Ibps So 2018 Previous Year Question Paper - It Officer Agriculture Field Officer HR Marketing Officer English E95c2eff

You might also like

Download as pdf or txt
Download as pdf or txt
You are on page 1of 52

 Useful Links

IBPS SO 2018
Previous Year Question
Paper – IT Officer,
Agriculture Field Officer,
HR, Marketing Officer
English

1
Direction: Find the wrong term in 7. 2x2 + 17x + 30 = 0,
the given series: II.4y2 – 7y – 15 = 0
1. 8, 9, 20, 64, 256, 1285, 7716 A. X > Y B. X < Y
A. 1285 B. 256 C. X ≥ Y D. X ≤ Y
C. 7716 D. 64 E. X=Y or No relation can be
E. 9 established

Direction: Find the wrong term in Direction: In the following question


the given series: two equations are given in variables
X and Y. You have to solve these
2. 1015, 508, 255, 130, 66.5, 35.75, equations and determine relation
20.875 between X and Y.
A. 1015 B. 20.875 8. 5x2 + 7x – 6 = 0,
C. 66.5 D. 255 II. 3y2 + 19y + 28 = 0
E. 130 A. X > Y B. X < Y
C. X ≥ Y D. X ≤ Y
Direction: Find the wrong term in E. X=Y or No relation can be
the given series: established
3. 7, 4, 5, 9, 20, 51, 160.5
A. 4 B. 5 Direction: In the following question
C. 7 D. 51 two equations are given in variables
E. 160.5 X and Y. You have to solve these
equations and determine relation
Direction: Find the wrong term in between X and Y.
the given series. 9. I. 8x2 - 6x – 5 = 0,
4. 1108, 1117, 1142, 1191, 1312, 1480 II. 2y2 - 7y – 4 = 0
A. 1117 B. 1108 A. X > Y B. X < Y
C. 1191 D. 1480 C. X ≥ Y D. X ≤ Y
E. 1142 E. X=Y or No relation can be
established
Direction: Find the wrong term in
the given series. Direction: In the following question
5. 46 , 47 , 55 , 84 , 146 , 271 two equations are given in variables
A. 47 B. 271 X and Y. You have to solve these
C. 46 D. 84 equations and determine relation
E. 55 between X and Y.
10. I. 6x2 + 5x - 4 = 0
Direction: In the following question II. y(10y – 11) = –3
two equations are given in variables A. X > Y B. X < Y
X and Y. You have to solve these C. X ≥ Y D. X ≤ Y
equations and determine relation E. X=Y or No relation can be
between X and Y. established
6. I. x2 - 6x = 7
II. 2y2 + 13y + 15 = 0 Direction: Given below are two
A. X > Y B. X < Y quantities named I and II. Based on
C. X ≥ Y D. If X ≤ Y the given information, you have to
E. X=Y or No relation can be determine the relation between the
established two quantities. You should use the
given data and your knowledge of
Direction: In the following question Mathematics to choose among the
two equations are given in variables possible answers.
X and Y. You have to solve these 11. Quantity I: A tap can fill a tank in 15
equations and determine relation sec. But due to leakage in the
between X and Y. bottom; it is filled in 20 sec. If the

2
tank is full, what amount of time is
required to empty it completely? Direction: Given below are two
Quantity II: 50 sec quantities named I and II. Based on
A. Quantity I > Quantity II the given information, you have to
B. Quantity I < Quantity II determine the relation between the
C. Quantity I ≥ Quantity II two quantities. You should use the
D. Quantity I ≤ Quantity II given data and your knowledge of
E. Quantity I = Quantity II or No Mathematics to choose among the
relation possible answers.
14. Quantity I: A car covers a distance
Direction: Given below are two between A and B in 50 minutes. If the
quantities named I and II. Based on speed of the car is reduced by 9 km
the given information, you have to per hour then the same distance is
determine the relation between the covered in 56 minutes. The distance
two quantities. You should use the between A and B
given data and your knowledge of Quantity II: Value of Y: Y2 – 40Y -
Mathematics to choose among the 2100 = 0
possible answers. A. Quantity I > Quantity II
12. Quantity I: 18 years ago the ratio of B. Quantity I < Quantity II
A's age to B's age was 8:13. The C. Quantity I ≥ Quantity II
present age of the ratio of their ages D. Quantity I ≤ Quantity II
is 5:7. Find the age of A, 13 years E. Quantity I = Quantity II or No
ago? relation
Quantity II: the value of Y: Y2 +
32Y - 185=0 Direction: Given below are two
A. Quantity I > Quantity II quantities named I and II. Based on
B. Quantity I < Quantity II the given information, you have to
C. Quantity I ≥ Quantity II determine the relation between the
D. Quantity I ≤ Quantity II two quantities. You should use the
Quantity I = Quantity II or No given data and your knowledge of
relation Mathematics to choose among the
possible answers.
Direction: Given below are two 15. Quantity I: A book seller sold a book
quantities named I and II. Based on at Rs. 56 in such a way that his
the given information, you have to percentage profit is same as the cost
determine the relation between the price of the book. If he sells it at twice
two quantities. You should use the the percentage profit of its previous
given data and your knowledge of percentage Profit then the new
Mathematics to choose among the selling price will be?
possible answers. Quantity II: Value of Y: Y2- 95Y+
13. Quantity I: A bought a certain 1500=0
quantity of bananas at a total cost of A. Quantity I > Quantity II
Rs. 1500. He sold 1/3 of these B. Quantity I < Quantity II
bananas at 25% loss. If he earns an C. Quantity I ≥ Quantity II
overall profit of 10%, at what D. Quantity I ≤ Quantity II
percentage profit did A sell the rest of E. Quantity I = Quantity II or No
the bananas? relation
Quantity II:
Direction (16 – 21) : Table given
A. Quantity I > Quantity II below shows the total number of
B. Quantity I < Quantity II customers in four different cities and
C. Quantity I ≥ Quantity II number of customers who buy
D. Quantity I ≤ Quantity II neither laptop nor tablet and the
E. Quantity I = Quantity II or No percentage of customers who buys

3
laptop out of the number of data given in the statement is
customers who buy. sufficient for answering the question.
You should use the data and your
knowledge of Mathematics to choose
the best possible answer.
21. Train 'A' running at a certain speed
crosses another train 'B' running at a
Note: Customers buy either laptop certain speed in the opposite
or tablet. direction in 12 seconds. What is the
length of train 'B'?
16. Number of customers who buys I. The length of both the trains
laptop from Kanpur is how much together is 450 metres.
more or less than the number of II. The difference between the speed
customer who buys tablet from of both Trains is 10 km/hr.
Surat? A. the data in Statement I alone are
A. 25% B. 50% sufficient to answer the question,
C. 75% D. 100% while the data in Statement II alone
E. 125% are not sufficient to answer the
17. What is the total number of question
customers who buy either laptop or B. the data in Statement II alone are
tablet from all the cities together? sufficient to answer the question,
A. 1310 B. 1210 while the data in Statement I alone
C. 1340 D. 1290 are not sufficient to answer the
E. 1360 question.
18. If the number of customers who C. the data either in Statement I or
buys laptops and tablets from city in Statement II alone are sufficient to
Jabalpur are in the ratio 6 : x and answer the question
the number of customer who buys D. the data in both Statements I and
laptops from Jabalpur are 40% more II together are necessary to answer
than that from Kanpur and the question.
the number of customers who buys E. the data even in both Statements
tablets from Jabalpur are 30% less I and II together are not sufficient to
than that from Kanpur, then the answer the question.
value of (x + 3) is 22. What is the quantity of milk in 80
A. 10 B. 12 litres of mixture of milk and water?
C. 7 D. 13 I. If 8 litres of mixture is replaced by
E. 15 equal quantity of water the ratio of
19. What is the ratio of number of milk and water in the mixture
customers who buys any one gadget becomes 27:13 respectively.
from Surat to the number of II. If 16 litres of mixture is replaced
customer who do not buy any gadget by equal quantity of milk, the ratio of
from Delhi? milk and water in the mixture
A. 15: 2 B. 17: 4 becomes 4:1 respectively.
C. 11: 7 D. 19: 9 A. the data in Statement I alone are
E. None of these sufficient to answer the question,
20. What is the average number of while the data in Statement II alone
customers who buys laptop from are not sufficient to answer the
Delhi, Kanpur and Surat? question
A. 90 B. 91 B. the data in Statement II alone are
C. 92 D. 95 sufficient to answer the question,
E. 93 while the data in Statement I alone
are not sufficient to answer the
Direction: Each question below is question.
followed by two statements I and II.

4
C. the data either in Statement I or 24. What is the cost of painting two
in Statement II alone are sufficient to adjacent walls of a hall having no
answer the question door or window at Rs. 750 per m2?
D. the data in both Statements I and I. Length and breadth are in the ratio
II together are necessary to answer of 7:3 respectively.
the question. II. Perimeter of the hall is 150 m and
E. the data even in both Statements height is one-fifth of the perimeter.
I and II together are not sufficient to A. If the data in Statement I alone
answer the question. are sufficient to answer the question,
while the data in Statement II alone
Direction: Each question below is are not sufficient to answer the
followed by two statements I and II. question
You have to determine whether the B. If the data in Statement II alone
data given in the statement is are sufficient to answer the question,
sufficient for answering the question. while the data in Statement I alone
You should use the data and your are not sufficient to answer the
knowledge of Mathematics to choose question.
the best possible answer. C. If the data either in Statement I or
23. By how many years is Suresh older in Statement II alone are sufficient to
than his brother Rakesh? answer the question
I. Ratio between Rakesh’s present D. If the data in both Statements I
age and Suresh’s age after four years and II together are necessary to
is 5 : 7 respectively. answer the question.
II. Ratio between Rakesh’s age four E. If the data even in both
years ago and Suresh’s present age Statements I and II together are not
is 2 : 3 respectively. sufficient to answer the question.
A. If the data in Statement I alone
are sufficient to answer the question, Direction : Each question below is
while the data in Statement II alone followed by two statements I and II.
are not sufficient to answer the You have to determine whether the
question data given in the statement is
B. If the data in Statement II alone sufficient for answering the question.
are sufficient to answer the question, You should use the data and your
while the data in Statement I alone knowledge of Mathematics to choose
are not sufficient to answer the the best possible answer.
question. 25. Salary of A and B is in ratio 3:4 and
C. If the data either in Statement I or expenditure is in ratio 4:5. What is
in Statement II alone are sufficient to the ratio of their saving?
answer the question I. B’s saving is 25% of his salary.
D. If the data in both Statements I II. B’s salary is Rs. 2500 &
and II together are necessary to expenditure of A is 880 Rs.
answer the question. A. If the data in Statement I alone
E. If the data even in both are sufficient to answer the question,
Statements I and II together are not while the data in Statement II alone
sufficient to answer the question. are not sufficient to answer the
question
Direction: Each question below is B. If the data in Statement II alone
followed by two statements I and II. are sufficient to answer the question,
You have to determine whether the while the data in Statement I alone
data given in the statement is are not sufficient to answer the
sufficient for answering the question. question.
You should use the data and your C. If the data either in Statement I or
knowledge of Mathematics to choose in Statement II alone are sufficient to
the best possible answer. answer the question

5
D. If the data in both Statements I what percent of total passengers in
and II together are necessary to train C in all the coaches?
answer the question. A. 11% B. 21%
E. If the data even in both C. 31% D. 19%
Statements I and II together are not E. 29%
sufficient to answer the question. 27. Ratio of number of females to
number of males in train E (all the
Direction (26 – 31) : Study the coaches together) is 11: 24. Then
given data and answer the following find the difference of number of
questions. males in train E and AC passengers in
Pie chart given below shows the train A and D together?
percentage distribution of total A. 672 B. 562
number of passengers in all the C. 768 D. 548
coaches of five trains train A, train B, E. 692
train C, train D and train E. 28. Find the average no. of passengers
Total number of passengers who are not in the AC coach in train
travelling by the given five trains = A, C and D in all the coaches
8000 together.
A. 1000 B. 1008
C. 1208 D. 1200
E. 1288
29. What is the ratio of number of
passengers in AC coach in train B and
D together to the total number of
passengers in train A in all the
coaches.
A. 36: 53 B. 33: 50
C. 39: 47 D. 35: 41
E. 50: 37
30. Number of AC passengers in train C
and D together is what percent more
or less than number of passengers in
train D in all the coaches?
Pie chart given below shows the A. 54.5% B. 68.5%
percentage distribution of total C. 72.5% D. 62.5%
number of passengers in AC coaches 58.5%
of the five trains train A, train B, train 31. Amit and Ruchi started a business by
C, train D and train E. investing Rs 2000 and Rs 2800
Total number of passengers respectively. After 8 months, Amit
travelling in AC coaches of the given added Rs 600 and Ruchi added Rs
five trains = 1200 400. At the same time Roni joined
them with Rs 4200. Find the share of
Amit if they get a profit of Rs 15,680
after a year.
A. 4280 Rs B. 5280 Rs
C. 6630 Rs D. 7360 Rs
E. None of these
32. A 450 meters long train cross a girl
who is walking with a speed of 5
km/hr in the direction of the moving
train in 27 sec. find how long train will
cross a 525 m long platform?
26. Total passengers in AC coach in train
A. 54 sec B. 56 sec
A and C together is approximately
C. 48 sec D. 65 sec

6
E. None of these E. Rs. 521000
33. How many bricks are needed to 38. Find the ratio of sold products of
complete a wall 15 m × 12 m × 10 company P and T together to the sold
cm using bricks 24 cm × 25 cm × 10 products of company Q and S
cm thick if 37(1/2)% of the wall is together.
already built? A. 229: 299 B. 321: 341
A. 1560 B. 1750 C. 276: 299 D. 229: 268
C. 1875 D. 1965 E. None of these
E. None of these 39. Find the average no. of sold product
34. A In an election between two of companies P, Q, S and U.
candidates, one got 16(2/3)% more A. 2398.5 B. 2566.5
votes than other, 20% of votes were C. 2622.5 D. 2799.5
invalid. If the number of votes was E. 2456.5
7800, what was the number of valid 40. If the total production of company R
votes, Winner candidate got? and T are increased by 20% and 30%
A. 3480 B. 4230 respectively and the percentage of
C. 3360 D. 4650 unsold products is same. Find the
E. None of these sum of the sold product of both the
35. If a student walks from his house to companies.
school at 8 km/hr, he is late by 30 A. 1110.8 B. 2220.8
minutes. However, if he walks at 10 C. 2267.8 D. 2245.8
km/hr, he is late by 6 minutes only. E. 1154.8
The distance of his school from his 41. The average age of amisha and
house is. Aaradhana is 35 years. If Alka
A. 15 km B. 16 km replaces amisha, the average age is
C. 18 km D. 12 km 31 years, if Alka replaces Aaradhana
E. None of these average age is 36 years. If the
average age of Aditi and Aashirya is
Direction (36 – 40) : Bar chart half of average age of amisha,
shows the total production (in '00) of Aaradhana and Alka. then average
six different companies (P, Q, R, S, T, age of all the five people is
U) and unsold product (in %) of these A. 25.6 B. 27.2
companies. Study the graph carefully C. 26.8 D. 24.5
and answer the questions: E. None of these
42. The sum invested in Scheme B is
thrice the sum invested in Scheme A.
The investment in Scheme A is made
for 4 years at 8% p.a. simple interest
and in Scheme B for 2 years at 13%
p.a. simple interest. The total
interest earned from both the
schemes is Rs.1320. How much
36. Total production of company P and U amount was invested in A?
is how much more or less than total A. 1250 B. 1200
production of company R and T. C. 1180 D. 1050
A. 4200 B. 4000 E. None of these
C. 3800 D. 4400 43. A starts a business, after 4 months B
E. 3600 also join him with Rs.16000 and after
37. If the total revenue of company Q 6 months of B’s joining C also join
and S is Rs.1344000 and ratio of cost them with Rs.26000. If A gets
of product sold by company Q and S approx. Rs 3600 out of total annual
is 2: 3, find the revenue of company profit of Rs.8100. then find the
Q. approximate value of initial
A. Rs. 534000 B. Rs. 528000 investment of A.
C. Rs. 568000 D. Rs. 539000

7
A. Rs.10000 B. Rs.12000 E. None of these
C. Rs.15000 D. Rs.18000 48. Total students applied for P.O. are
E. None of these what percent more or less than total
44. A and B working alone can do a work students applied for S.O.
in 40 days and 30 days respectively. A. 15% B. 25%
They started the work together but B C. 35% D. 10%
left after sometime and A finished E. 40%
remaining work in 5 days. Find after 49. Male students applied for P.O. and
how many days from start B left the clerk together are what percent of
work? female students applied for clerk and
A. 15 days B. 14 days S.O. together.
C. 16 days D. 12 days A. 100% B. 50%
E. 18 days C. 125% D. 150%
45. Mukesh spends 20% of his monthly E. 200%
income on household expenditure. 50. Female students applied for P.O and
Out of the remaining 25% he spends S.O. together are how much more or
on children’s education, 15% on less than male students applied for
transport, 15% on medicine and 10% same posts.
on entertainment. He is left with Rs. A. 25/9 % B. 50/3 %
9800 after incurring all these C. 25/3 % D. 50/9 %
expenditures. What is his monthly E. None of these
income?
A. 45000 B. 36000 Direction (51 – 55) : In the
C. 32000 D. 35000 passage given below, there are
E. None of these blanks, each preceded by a word
given in bold. Every blank has four
Direction (46 – 50) : Study the alternative words given in options
passage given below and answer the (A), (B), (C) and (D). Find the word
following questions. which best suits the place. If the
Students applied for three posts in given word suits the blank, mark 'no
the IBPS examination - Probationary correction/change required' as the
officer, specialist officer and clerk. answer.
Male students applied for clerk are 3 owadays, globalization’s opponents
times the male students applied for seem increasingly to be drowning out
P.O. and male students applied for its defenders. If they get their way,
S.O. are 120 more than male student the post-World War II international
applied for P.O. Ratio of male to order – which aimed, often
female students applied for P.O. is 2: successfully, to advance peace and
3 and female students applied for prosperity through exchange and
S.O. and clerk are same. Male applied connection – could
for P.O. are 25% less than female well resemble (###Q1###). Can
applied for clerk. Total number of globalization be saved?
students applied for the examination At first glance, the outlook appears
is 1220. grim. Every aspect of globalization –
46. Find the total students applied for the free trade, free movement of capital,
post of Clerk. and international migration – is under
A. 520 B. 540 attack. Leading the charge
C. 420 D. 560 are couthy (###Q2###) forces –
E. 460 from populist political parties to
47. Find the ratio of total male students separatist groups to terrorist
to total female students applied in organizations – whose actions tend to
the examination. focus more on what they oppose than
A. 31: 25 B. 42: 19 on what they support.
C. 36: 25 D. 36: 23

8
In Russia and Asia, anti-Western C. facilitation D. compunctions
groups are at the forefront of the E. No change required
campaign against globalization. In 55. Choose the correct response from the
Europe, populist parties have tended given words.
to emphasize their aversion to A. assimilation B. distraction
European integration, with those on C. destruction D. institutions
the right often E. No change required
also condemning (###Q3###) i Direction: In this question,
mmigration, while the left denounces two columns I and II and three
rising economic inequality. In Latin sentences are given, which are
America, the enemy seems to be divided into two parts. Column I (A,
foreign abetment (###Q4###) o B and C) consists of first half of each
f any kind. In Africa, tribal separatists sentence and Column II (D, E and
oppose anyone standing in the way of F) consists of second half of each
independence. And in the Middle sentence. Match column I with
East, the Islamic State (ISIS) column II, so that the sentences
virulently rejects modernity – and formed are both meaningful and
targets societies that embrace it. grammatically correct. Choose the
Despite their differences, these option as your answer.
groups have one thing in common: a 56. I.
deep hostility toward international A) The aim appears to be to capture
structures and interconnectedness a vote bank without pondering over
(though, of course, a murderous B) The Bill falls short of the
group like ISIS is in a different government’s own laudable claims
category from, say, European and objectives
populists). They do not care that the C) Our leaders and some
international order they want to tear academicians seemed obsessed with
down enabled the rapid post-1945 our glorious past
economic growth that liberated II.
billions of developing-country D) of providing shelter to the
citizens from poverty. All they see are minorities in India’s neighbourhood
massive, on at least two counts.
unbending organisation (###Q5# E) of esoteric beliefs which augur
##) and intolerable inequalities in doom for the science.
wealth and income, and they blame F) the after-effects on other Indian
globalization. citizens.
51. Choose the correct answer from the A. B-D B. A-E and C-F
given options. C. C-E
A. collapse B. succeed D. A-E, B-F and C-D
C. rise D. fluctuate E. B-D and A-F
E. No change required
52. Choose the correct option from the Direction: In this question,
given words. two columns I and II and three
A. antagonistic B. chummy sentences are given, which are
C. pally D. providential divided into two parts. Column I (A,
E. No change required B and C) consists of first half of each
53. Choose the correct word from the sentence and Column II (D, E and
given options. F) consists of second half of each
A. complimentary B. flattering sentence. Match column I with
C. absolving D. laudatory column II, so that the sentences
E. No change required formed are both meaningful and
54. Choose the correct word from the grammatically correct. Choose the
given options. option as your answer.
A. compaction B. interference 57. I.

9
A) The preposterous claims such as Direction: In this question,
some of those made in the Science two columns I and II and three
Congress are disheartening sentences are given, which are
B) People from a Hindu background divided into two parts. Column I (A,
have not only faced denial of rights B and C) consists of first half of each
and violence sentence and Column II (D, E and
C) Equality before the law, economic F) consists of second half of each
justice and secularism are being sentence. Match column I with
whittled down one by one column II, so that the sentences
II. formed are both meaningful and
D) which delineate the image of sane grammatically correct. Choose the
voices being outshouted and correct option as your answer.
flourishing of divisive politics. 59. I.
E) howbeit, congress party seems to A) The current President of the
launch a few new potential United states
candidates for the coming elections. B) The amount of time it takes to go
F) but also generations of hatred to the store
between the two communities are C) Most players don’t remain fit
well known. II.
A. B-D and C-D B. A-E and C-F D) To play for a long time.
C. C-D D. A-E and B-F E) Is very outspoken and
E. C-D and A-F unapologetic.
F) For the stars in the universe.
Direction: In this question, A. A-E and B-D B. C-E
two columns I and II and three C. A-E and C-D
sentences are given, which are D. B-D, A-E and C-F
divided into two parts. Column I (A, E. B-F
B and C) consists of first half of each
sentence and Column II (D, E and Direction: In this question,
F) consists of second half of each two columns I and II and three
sentence. Match column I with sentences are given, which are
column II, so that the sentences divided into two parts. Column I (A,
formed are both meaningful and B and C) consists of first half of each
grammatically correct. Choose the sentence and Column II (D, E and
option as your answer. F) consists of second half of each
58. I. sentence. Match column I with
A) A man died shortly after he was column II, so that the sentences
arrested formed are both meaningful and
B) If there is a single word that grammatically correct. Choose the
describes what Christmas is all about correct option as your answer.
C) The inner ear is so sensitive that it 60. I.
can detect A) He was one of the best players in
II. the world
D) prompting an urgent investigation B) Bugatti created one of the
into the incident. C) The pencil was lying on the floor
E) distinguish the sounds that are when
only 10 millionths of a second apart. II.
F) it is a Hebrew word, “Emmanuel” D) The cat came running and tripped
founded in the gospel of Matthew. over it.
A. B-D B. A-D and B-F E) But his excellence seems to have
C. C-E deserted him recently.
D. A-E, B-F and C-D F) Very well done for that matter.
E. C-D and A-F A. A-F and C-D B. C-F and B-E
C. A-E and C-D D. B-F

10
E. B-D C. C D. E
E. F
Direction: Given below are six 64. Which of the following will be the fifth
statements A, B, C, D, E and F, which statement after rearrangement?
when arranged in the correct order, A. A B. B
form a coherent and meaningful C. C D. E
paragraph. The sentence marked D is E. F
fixed and would fit in the fourth 65. Which of the following will be the last
position. Rearrange the other statement after rearrangement?
statements in a proper sequence to A. A B. B
form a meaningful paragraph, then C. C D. E
answer the questions that follow. E. F
A) While an inquiry is being
conducted, it is suspected that the Direction: In the given question,
deaths may have occurred due to five words are printed in bold and are
infighting and disease according to marked A, B, C, D and E. The
reports. positions of some highlighted words
B) If we don’t check our misuse of may be incorrect and need to be
natural resources more and more exchanged with another highlighted
such appalling news will keep word to make the sentence correct.
surfacing from time to time. Find the pair of word(s) that need to
C) Earlier this week, Gujarat’s be exchanged.
treasured wildlife reserve suffered a 66. More (A) than a decade ago,
shocking jolt in the form of mass lion Indic fonts (B) were primarily
deaths. being designed (C)
D) While some have presumably died by software (D) developers
in infights, others, including three who sold (E) them as supporting
lion cubs, are suspected to have products.
contracted a fatal illness. A. A-C B. C-D & B-C
E) Infighting in the wild mostly occurs C. C-E D. A-B & C-E
when natural resources are scarce E. No exchange required
and animals encroach upon each
other's territory for basic necessities. Direction: In the given question,
F) 11 lion carcasses were reportedly five words are printed in bold and are
found in the East division of Gir forest marked A, B, C, D and E. The
and the deaths have presumably positions of some highlighted words
taken place within 10 days as stated may be incorrect and need to be
by east Gir’s Deputy Conservator of exchanged with another highlighted
Forest, P Purushothama. word to make the sentence correct.
61. Which of the following will be the first Find the pair of word(s) that need to
statement after rearrangement? be exchanged.
A. A B. B 67. The migrations (A) of the Partition
C. C D. E of India led to the tragic death (B)
E. F and displacement of millions of
62. Which of the following will be the people in what was to become (C)
second statement after one of the greatest event (D)
rearrangement ? in history (E).
A. A B. B A. A-D B. C-D & B-C
C. C D. E C. C-E D. A-B & C-E
E. F E. No exchange required
63. Which of the following will be the
third statement after rearrangement Direction: In the given question,
? five words are printed in bold and are
A. A B. B numbered A, B, C, D and E. The

11
positions of some highlighted words C. C-D
may be incorrect and need to be D. B-E
interchanged with another E. A-C and D-E
highlighted to make the sentence
correct. Find the word(s) that need to Direction (71 – 77) : Read the
be exchanged. given passage carefully and answer
68. India has to prepare (A)/ for the questions that follow. Certain
the consequences (B)/ of climate words are printed in bold to help you
change, with risk (C)/ early warning locate them while answering some of
systems, engineering (D)/ to cope these.
with the fallout and better (E)/ The Financial Times with a survey
transfer mechanism. concluded that millennials are
A. C-E B. B-C prioritizing short-term spending over
C. C-D & A- E long-term saving. According to one
D. No exchange required calculation, the average 25-year-old
E. B-E & A-D should be saving £800 (or about
$1,146) a month over the next 40
Direction: In the given question, years, in order to retire at 65 with an
five words are printed in bold and are annual income of £30,000. That piece
numbered A, B, C, D and E. The went viral. For all the wrong reasons.
positions of some highlighted words As millennials who responded angrily
may be incorrect and need to be to the article noted, they’re too busy
interchanged with another buying groceries or paying rent to
highlighted to make the sentence even think about being able to have
correct. Find the word(s) that need to that much money to allocate to a
be exchanged. savings account. But ignoring the
69. Companies (A)/ with high tone deafness, there is a real
market share (B)/ often receive problem here.
better prices from suppliers (C), as T Rowe Price recommends that
their large (D)/ order volumes millennials should save about 15% of
increase their purchasing (E)/ their incomes for retirement.
power. However, a recent survey found that
A. A-B and C-D B. A-B on average, while they are doing a
C. A-C and B-D good job of budgeting and say they
D. No exchange required have increased their savings in the
E. A-B and C-E past 12 months, their actual savings
rate is about 8%. Financial planners
Direction: In the given question, can puff and huff about results like
five words are printed in bold and are that. They can argue that millennials
numbered A, B, C, D and E. The don’t realize how much they need to
positions of some highlighted words save; that they are succumbing to
may be incorrect and need to be one of those behavioural finance
interchanged with another phenomena by failing to appreciate
highlighted to make the sentence that yes, one day they, too, will be 65
correct. Find the word(s) that need to and need a retirement nest egg. For
be exchanged. their part, the millennials might well
70. If the world factoring (A) to pursue argue that the rest of us simply don’t
the current model (B) of economic understand their new normal.
growth without continues (C) in It has always been true, and remains
environmental costs, one million true today, that a dollar someone
species could go decades (D) in a puts aside in a tax-sheltered
matter of extinct (E). retirement account when he is 25
A. D-E years old will be worth much, much
B. No exchange required. more then that same dollar would be

12
if he had set it aside at the age of 50, that millennials can’t start saving for
thanks to the fact that it is sitting retirement when it’s most
there are being reinvested, year after advantageous for all of society that
year, tax-free. What someone in their they should, maybe there’s a way to
20s loses in absolute wealth, they restructure or postpone debt
earn in terms of time. The problem is payments until later in life as long as
that there are too many other factors graduates begin contributing to their
stopping millennials from making retirement accounts.
that decision to save. Source:
They have got the millennial https://www.theguardian.com
paradox to contend with. In 2014, 71. What does the author mean by
the average college student “...They have got the millennial
graduated with $33,000 of student paradox to contend with”?
debt, according to one calculation. Do A. Millennials are contradictory,
you want to be one of those students counterintuitive, or downright
who defaults on her student debt, confusing.
just in order to have a few extra B. Millennials have to struggle with
bucks to put into her retirement the fact that they are a generation
account? Really, not a viable packed with contradictions.
solution; those payments have to be C. Millennials are struggling with all
kept up, even if it means there’s no the competing demands on their
money for a retirement account. The dollars, just when they know that if
cost of living is climbing, too, led by they put them to work in their
rental costs, which hit records in retirement account, they’d do them
many cities last year. On average, the most good.
millennials who rent nationwide D. Millennials are dealing with the
would have had to spend 30% of fact that they are in their 20s and are
their monthly income to their actually making money, but the
landlords. Health insurance? If your problem is that something or
company offers it, odds are it’s a someone is after every single dollar.
benefit that requires you to shoulder E. Millennials got to contend with the
a greater portion of the costs of these fact that they are being both fully
days. And if you’re older than 26, and aware of saving for the future, and
paying for your own healthcare, also they are being completely
you’ve already discovered that both ignorant of it at the same time.
premiums and deductibles are rising 72. Which of the following is either a
for most policies. Then, there are synonym or an antonym of the
other expenses like wedding gifts, highlighted word used in the
birthday gifts, parties, clothes, passage?
accessories, food, bills, ABATE
transportation, travel or vacations. I. Wane II. Recede
Eventually, some of those pressures III. Prolong
will abate – the student debt will be A. Only III B. Both I and III
paid down – and millennials will be C. Both II and III
earning more. But they will be older, D. Both I and II
and the value of each dollar they save E. All of these
by that stage will be less. In any 73. Which of the following is not an
event, new financial pressures will assumption that supports the
arrive on the scene, in the form of arguments presented in the third
children, the need to save for a paragraph?
house, to help out ageing parents. A. Retirement is expensive.
Perhaps there is some creative way B. Investing a small amount early
to tackle this. To the extent that the resulted in more retirement earnings
cost of obtaining an education means

13
than investing a much larger amount B. But ignoring the tone deafness,
later. there is a real problem here.
C. Saving early is the easiest way to C. However, a recent survey found
accumulate money because time is that on average, while they are doing
on your side. a good job of budgeting and say they
D. Because of compound interest, have increased their savings in the
the money you contribute now is very past 12 months, their actual savings
valuable. rate is about 8%.
E. A tax-sheltered retirement D. Financial planners can puff and
account allows you to defer tax huff about results like that.
payments on any returns earned E. They can argue that millennials
within the retirement account until don’t realize how much they need to
you start making withdrawals from save; that they are succumbing to
them (usually, after retirement). one of those behavioural finance
74. Given below is a possible inference phenomena by failing to appreciate
that can be drawn from the facts that yes, one day they, too, will be 65
stated in the fourth paragraph. You and need a retirement nest egg.
have to examine the inference in the 77. Which of the following correctly
context of the passage and decide describes the tone of the last
upon its degree of truth or falsity. paragraph?
'The lack of saving culture will impact A. Sanguine B. Constructive
on the consumption patterns for C. Sarcastic D. Satirical
millennials.' E. Inspiring
A. Definitely true
B. Probably true Direction: The following question
C. The data are inadequate carries two statements with one
D. Probably false blank each. Choose the word which
E. Definitely false would fit in both the blanks.
75. Given below is a possible inference 78. 1) Their teacher was on leave, so
that can be drawn from the facts they had a day's _______ before
stated in the fourth paragraph. You their essays were due.
have to examine the inference in the 2) The organisation had
context of the passage and decide no ________ from the demands of
upon its degree of truth or falsity. the workers.
'The financial struggles A. Benefit B. Censure
that millennials face are making C. Respite D. Advantage
them more financially savvy.' E. Breaks
A. Definitely true
B. Probably true Direction: Two sentences with one
C. The data are inadequate blank in each, followed by five
D. Probably false alternatives, are given. Choose that
E. Definitely false option as the answer which can fill
76. Given below are five statements from both the blanks of both the sentences
the first and the second paragraph. making it grammatically correct and
Choose the statement which is meaningful.
grammatically or contextually 79. A) It is only by observing people in
incorrect. _________ situations that we come
A. According to one calculation, the to know each other.
average 25-year-old should be B) According to the Shinto doctrine,
saving £800 (or about $1,146) a spirits of the dead can act upon the
month over the next 40 years, in _________ world.
order to retire at 65 with an annual A. Troubled B. Gruesome
income of £30,000. C. Desolate D. Mundane
E. Exultant

14
There may be an error in one or more
Direction: Two sentences with one parts. Determine the part(s) which
blank in each, followed by five requires correction and mark it as
alternatives, are given. Choose that your answer.
option as the answer which can fill 83. Every motivation that every (I)/
both the blanks of both the sentences human being has is for the desire for
making it grammatically correct and pleasure or (II)/ it is a reaction of the
meaningful. frustration of not getting pleasure.
80. A) Madhurima found ________ in the (III)
childhood pictures of her little boy as A. Only I B. Only II
she turned the pages of the album. C. Only III D. Both I and II
B) After her rough breakup, the E. Both II and III
soundlessness of nature impressed
her and gave _________. Direction: A sentence divided into
A. Solace B. Forsaken three parts (I, II and III) is given.
C. Languid D. Gloomy There may be an error in one or more
E. Robust parts. Determine the part(s) which
requires correction and mark it as
Direction: Two sentences with one your answer.
blank in each, followed by five 84. He fed his goats to keep (I)/ them
alternatives, are given. Choose that alive but he fed the wild goats with
option as the answer which can fill plenty (II)/ of food to tempt them
both the blanks of both the stay back. (III)
sentences. A. Only I B. Only II
81. 1) At the time of the war, the spies C. Only III D. Both I and II
would often exchange information E. Both II and III
during a _____ in the church.
2) My mother-in-law and I will have Direction: A sentence divided into
a ______ in order to plan my three parts (I, II and III) is given.
husband’s surprise birthday party. There may be an error in one or more
A. Rescind B. Regression parts. Determine the part(s) which
C. Restitution D. Rendezvous requires correction and mark it as
E. Welter your answer.
85. Hundreds of thousands of protesters
Direction: Two sentences with one gathered in cities (I)/ into the U.S.
blank in each, followed by five last weekend for rallies organized
alternatives, are given. Choose that (II)/ in the wake of a deadly mass
option as the answer which can fill shooting at a high school in Parkland,
both the blanks of both the Florida. (III)
sentences. A. Only II B. Only I
82. I) The _____________ protests from C. Only III D. Both I and III
the residents made it certain that the E. Both I and II
new laws about the traffic control
would not be accepted easily. Direction: A sentence divided into
II) Two horses went three parts (I, II and III) is given.
_______________ when the There may be an error in one or more
president arrived to check the parts. Determine the part(s) which
parade. requires correction and mark it as
A. Uncontrolled B. Mellow your answer.
C. Rampant D. Erect 86. The task of self-realization would not
E. Obsolete be achieved through (I)/ haste or
heedlessness as it needs patience,
Direction: A sentence divided into enthusiasm, (II)/ attention and
three parts (I, II and III) is given. freedom from tension. (III)

15
A. Only I B. Only II sportspersons. There is no reason
C. Only III D. Both I and II why they should be considered for
E. Both II and III the Padma Awards at all.
I. When
Direction: A sentence divided into II. No sooner do
three parts (I, II and III) is given. III. As
There may be an error in one or more A. Only I B. Both I and II
parts. Determine the part(s) which C. Both II and III D. Both I and III
requires correction and mark it as E. All of the above
your answer.
87. Mullah Nasruddin was asked if he Direction: Select the phrase/
remembers any of the sayings (I)/of connector from the given options
the Prophet as being exceptionally which can be used to form a single
meaningful to him and he replied sentence from the two sentences
(II)/ that he knew a tradition retold given below, implying the same
by Akarma, which nobody else had meaning as expressed in the
heard. (III) statement sentences. Pick out the
A. Only I B. Only II option which when used to start a
C. Only III D. Both I and II sentence combines both the above
E. Both II and III sentences in one.
90. Organised sector generated few jobs
Direction: Select the phrase/ in India. Industries prefer capital-
connector from the given three intensive production despite the
alternatives which, when used at the economy’s relative abundance of
beginning and the middle of the two low-wage labour.
sentences, combines them into one. (I) Because organised sector
The sentence thus formed should generate few jobs
imply the same meaning as (II) Besides of abundance of low-
expressed in the two sentences. wage labour
88. After the announcement of periodic (III) Few organised sector jobs get
labour force surveys, India has not generated in India because
just created an opportunity for itself A. Only (I) B. Only (II)
to undo its recent lukewarm C. Only (III)
performance on the job creation D. Only (I) and (III)
front. It has also opened a lucrative E. None of these
window to make job creation the
heart of policy formulation. Direction: Select the phrase/
I. While.... being opened connector from the given options
II. With the announcement ... but which can be used to form a single
has also sentence from the two sentences
III. During... to withhold given below, implying the same
A. Only I B. Only II meaning as expressed in the
C. Only III D. Both I and III statement sentences. Pick out the
E. None of these option which when used to start a
sentence combines both the above
Direction: Select the phrase/ sentences in one.
connector from the given three 91. Officials of the forest department are
alternatives which, when used at the not able to identify medicinal plant
beginning, combines them into one. species. They are not given any
The sentence thus formed should special training about these plants.
imply the same meaning as (I) Identification of medicinal plant
expressed in the two sentences. species
89. There are so many prestigious (II) As the officials of the forest
awards exclusively for the department

16
(III) Because of special training 4 percent of the employment growth
A. Only (I) since 2013. Despite this economic
B. Only (II) pressure, rural America remains one
C. Only (III) of our nation’s most fertile regions,
D. Only (I) and (III) and recent advances in biotechnology
E. None of these are making it easier than ever to
sustainably grow new kinds of
Direction: Select the phrase/ valuable goods, from
connector from the given options biopharmaceuticals to biomaterials.
which can be used to form a single With the right strategic investments,
sentence from the two sentences rural America could see a biotech
given below, implying the same "bloom." A Bio-Belt stretching
meaning as expressed in the through middle America must be built
statement sentences. Pick out the to bring new skills and high-paying
option which when used to start a jobs to communities that desperately
sentence combines both the above need them. This initiative would
sentences in one. bolster investment in biotechnology
92. Nehru is hardly ever acknowledged training, education, infrastructure
as a decisive leader. There is and entrepreneurship in rural areas
overwhelming evidence of him being in order to develop new, sustainable
so. sources of income. The Bio-Belt is
I) Despite there being overwhelming about much more than biofuel.
evidence Fermentation is an increasingly
II) Since there is overwhelming powerful force for converting sugar
evidence and other forms of biomass into
III) Though there is overwhelming value-added goods—all through the
evidence rational design of cells that can be
A. Only I B. Only II sustainably grown wherever land is
C. Only III D. Only I and III abundant. Rural biotechnology, like
E. None of these all biotechnology, will require
strategic partnerships between
Direction (93 – 102) : Read the business and academia. Agriculture
given passage carefully and answer extension schools, which are often
the questions that follow. Certain located in rural areas, should expand
words are printed in bold to help you to deliver research and job training
locate them while answering some of for rural communities. Success will
these. also depend
Cow-free burgers are now all the on _______________ community
rage. That’s great for California, but colleges and local businesses to
rural America is stuck in a bind. provide a pipeline of individuals with
Technology is wiping out traditional the skills needed to work in regional
jobs, and high-tech training can be biotech clusters. The government
hard to come by in towns where should incentivise these
livestock outnumber people. Young partnerships. National laboratories
Americans know this. According to a dedicated to biological manufacturing
recent report from the United States are also needed in rural areas. These
Department of Agriculture Economic could be standalone new labs or
Research Service, non- expansions of existing facilities.
metropolitan communities are Biotechnology startups could use
greying as they attract retirees them as incubators to test their
and lose new members of the products without having to make a
labour force. Although rural major investment in equipment
communities are home to 14 percent themselves. At present, this vehicle
of the population, they have seen just for commercial success is mostly

17
limited to the coasts. Likewise, entrepreneurship with reference to
access to existing fermentation biotechnology in rural areas.
capacity should expand to those A. Only a B. Only b
outside of coastal cities. This could be C. Both a and b D. All a, b and c
one of the cheapest ways to spread E. Both b and c
the benefits of biotechnology to more 95. Which of the following words can
entrepreneurs. Under the Bio-Belt replace the phrase “wiping out” as
initiative, innovation would grow in used in the passage?
rural areas, and biomanufacturing A. Picturising B. Erasing
could expand across the country, C. Cleansing D. Sanitizing
where land and feedstock are E. None of the above
abundant. With new ideas, creative 96. Which of the following statement(s)
policies, supportive infrastructure is/ are true with reference to the
and sound investments in biotech context of the passage?
research and development, the a) The Bio-Belt initiative aims to
budding companies will be just the promote biomanufacturing across the
first of many to take root between the country, where land and feedstock
coasts of this great nation. As the are abundant.
country thinks about its future b) Fermentation acts as a tool to
manufacturing competitiveness and convert biomass into value added
infrastructure, let's remember this: goods.
21st-century infrastructure is not just c) The establishment of agricultural
roads and bridges. A critical research institutions in the rural
component of our economic growth areas will boost the Bio- Belt
will be the bioeconomy. initiative.
Source: https://www.forbes.com/ d) The author of the passage believes
93. What is the central theme of the that in the years to come,
passage? bioeconomy would be one of the
A. The future of biotechnology is major contributors to economic
bright in the urban America. growth of America.
B. Biotechnology will require a lot of A. Only a B. Only c
investment to thrive and hence, the C. Both a and d D. Only a, b and c
possibility of utilising it in the near E. All a, b, c and d
future is doubtful. 97. What can be understood from the
C. Strategic investment in following lines of the passage?
biotechnology can reinvigorate rural “…non-metropolitan
America and help secure the nation’s communities are greying as they
high-tech competitive advantage. attract retirees and lose new
D. The skilled labour in biotechnology members of the labour force…”
resides in the cities, thus, the cost of A. Off late, the non- metropolitan
executing biotechnology in the rural areas have been experiencing
areas will be quite high. economic slowdown when compared
E. None of the above. to the urban areas.
94. How would the Bio-Belt be B. The absence of the young in the
advantageous to the rural non- metropolitan communities is
communities? rendering the households gloomy.
a) The partnership with local C. Non- metropolitan areas are
businesses would boost the drawing the old and the retired
International business. population from and sending its
b) It would provide new skills and young population to the metropolitan
jobs to people who badly need them. areas, thus increasing the number of
c) It would strengthen training, the old in the former.
education, infrastructure and

18
D. Old people in the metropolitan 102.How many persons sit between Q
areas have become a burden over the and J, when counted from the right of
young. Q?
E. None of the above A. One B. Two
98. Which of the following phrases will fill C. Three D. Four
the blank mentioned in the passage? E. Five
A. Amalgamation among 103.Who sits opposite to F?
B. Partnerships between A. I B. J
C. Degeneration of C. P D. Q
D. Whether the emancipation of E. None of these
E. All the above 104.Four of the following five are alike in
99. What does the phrase “stuck in a a certain manner and hence form a
bind” mean? group. Find the one who does not
A. Glued to a pleasant situation belong to that group?
B. Caught in a difficult or awkward A. H B. Q
position. C. K D. I
C. Caught in a criminal offence E. J
D. In an agonizing dilemma 105.Who sits to the immediate left of P?
E. Released from a chaotic A. G B. Q
environment C. J D. H
100.Which of the following words can E. None of these
replace the word “incentivise” as
used in the passage? Direction (106 – 107) : Study the
A. Fund B. Capture following information carefully to
C. Promote D. Increase answer the questions.
E. Add 1) A + B means A is the brother of B.
2) A × B means A is the father of B.
Direction (101 – 105) : Read the 3) A ÷ B means A is the mother of B.
following information carefully to give 106.Which of the following would mean "P
the answer of the questions given is the son of Q"?
below: A. Q × R × P B. Q + P × R
Eight people F, G, H, I, J, K, P and Q C. Q ÷ P ÷ R D. Q × P + R
are seated around a circular table. E. None of the above
Among them, only three are facing 107.How is R related to Q if the given
the centre of the table and rests are relation is "Q + P ÷ R" ?
facing away from the centre. A. Uncle B. Son
F sits second to the right of H. G is C. Niece D. Nephew
not an immediate neighbour of both E. Cannot be determined
H and F. One of the immediate
neighbours of H faces the same Direction: In each of the following
direction as H. Q sits third to the right questions assuming the given
of F. K sits second to the right of Q. I statements to be true, find which of
does not sit next to G. P sits third to the two conclusions I and II given
the right of I. J and G face the same below is/are definitely true.
direction as I faces. Neither H nor F 108.Statement: P > Q, S < T, P = S
sits opposite to G. Conclusions:
101.What is the position of K with respect I. P = Q
to I? II. P > Q
A. Second to the left A. if only conclusion I is true
B. Second to the right B. if only conclusion II Is true
C. Immediate left C. if either conclusion I or II is true
D. Immediate right D. if neither conclusion I nor II is true
E. None of these E. if both conclusions I and II are true

19
Direction: In each of the following I. V < U
questions assuming the given II. L ≥ S
statements to be true, find which of A. Only conclusion I is true.
the two conclusions I and II given B. Only conclusion II is true.
below them is/are definitely true. C. Either conclusion I or II is true.
109.Statement: S < T, T > R, T = W D. Neither conclusion I nor II is true.
Conclusions: E. Both conclusions I and II are true.
I. R < S Directions (113 – 114) : In each of
II. S > W the following questions, a statement
A. if only conclusion I is true is followed by two course of action. A
B. if only conclusion II Is true course of action is taken for
C. if either conclusion I or II is true redressal, follow up, etc. Read the
D. if neither conclusion I nor II is true statement carefully and answer the
E. if both conclusions I and II are true question.
113.Statement: Although various efforts
Direction: In each of the following have been made over the past 10
questions assuming the given years and more recently, over the
statements to be true, find which of past couple of years in setting up the
the two conclusions I and II given infrastructure in various corners of
below them is/ are definitely true. the country to provide electricity,
110. Statement: X = Y < Z > W vast stretches of the country still
Conclusions: remain devoid of power.
I. Z = X Courses of Action:
II. Z > X I. The infrastructure should not only
A. if only conclusion I is true be installed but also set to use and
B. if only conclusion II Is true maintained and monitored by setting
C. if either conclusion I or II is true up of a National Energy Inspection
D. if neither conclusion I nor II is true body.
E. if both conclusions I and II are true II. The government should draft a
proposal for 100% FDI in power
Direction: In each of the following generation and supply.
questions assuming the given A. Only course of action I follows
statements to be true, find which of B. Only course of action II follows
the two conclusions I and II given C. Either I nor II follows
below is/are definitely true. D. Neither I nor II follows
111.Statement: E. Both I and II follow
S>E>C>U=R<E
Conclusions : 114.Statement: Although there is a
I. R < S much more hands-on approach to
II. C ≥ E teaching in the United States of
A. Only conclusion I is true. America, there is a deplorable lack of
B. Only conclusion II is true. IQ skill development which leads to a
C. Either conclusion I or II is true. very low level of knowledge among
D. Neither conclusion I nor II is true. school children about what happens
E. Both conclusions I and II are true. outside of the United States.
Courses of Action:
Direction: In each of the following I. There has to be an overhaul of the
questions assuming the given education system with an equal focus
statements to be true, find which of on skill development and knowledge
the two conclusions I and II given development.
below is/are definitely true. II. There needs to be an increase in
112.Statement : the number of general knowledge
V>A<L=U>E>S lectures by making them compulsory
Conclusions : for all schools.

20
A. Only course of action I follows E. None of the above
B. Only course of action II follows 119.Which among the following
C. Either I nor II follows statement is/are true?
D. Neither I nor II follows A. Student A becomes the head of the
E. Both I and II follow class on Tuesday and he has 18 roll
number
Directions (115 – 119) : Study the B. On Friday, Student F who has the
information carefully and answer the roll number of 24 becomes the head
questions given below. of the class
Six students A, B, C, D, E and F of C. Student B has the roll number of
Class 10 has been chosen to be the 18
head of the class for different days of D. The highest roll number belongs to
the week starting from Monday to the student who is the head of the
Saturday. The roll number of the class on Wednesday
students are 17, 18, 20, 21, 23 and E. None of the above
24 not necessarily in the same order.
The head of the class on Wednesday Direction (120 – 122) : Study the
is Student D and on Saturday is B. following information carefully and
The student whose roll number is 20 answer the questions given below:
becomes the head of the class on Six colleagues Laksh, Bhoomika, Sia,
Thursday. Student C becomes the Dravid, Esha and Palak are working
head of the class just after student A. in the same office and take different
The student with roll number 24 is time to reach his/her office. All of
the next student to become the head them take time in the multiples of ten
of the class after student C. The head in such a manner that the one who
of the class on Monday is neither reaches office the earliest, reaches in
Student E nor F. The difference of roll 10 minutes and the one who takes
number of student B and student F is maximum time reaches office in 60
4. The difference between the roll minutes. Dravid takes more time only
number of the head of the class on than Esha but less time than Laksh.
the first day of the week and the last Laksh reaches in 30 minutes.
day of the week is 1. Bhoomika takes less time than only
115.The student D becomes the head on Palak.
the class on which day of the week? 120.How much time does Sia take to
A. Thursday B. Friday reach office?
C. Wednesday D. Tuesday A. 60 minutes B. 50 minutes
E. None of the above C. 40 minutes D. 20 minutes
116.What is the name of the student who E. Cannot be determined
has 20 roll number and becomes the 121.Who amongst the following takes
head of the class on Thursday? maximum time to reach office?
A. Student E B. Student D A. Bhoomika B. Sia
C. Student F D. Student A C. Dravid D. Palak
E. None of the above E. Cannot be determined
117.What is the sum of the roll number of 122.How many people take more time to
students who becomes the head of reach office than Dravid?
the class on Friday and Saturday? A. Four B. Three
A. 41 B. 44 C. Two D. One
C. 47 D. 38 E. None
E. None of the above
118.What is the roll number of the Direction (123 – 127) : Study the
student C who becomes the head of following information carefully to
the class on Tuesday? answer the given questions.
A. 18 B. 23 In a certain code language,
C. 17 D. 21

21
‘saurabh major go army’ is written as E. If both I and II are implicit
‘tik mik pik sik’.
‘army convoy go further’ is written as Direction: In the question below, a
‘gik tik cik mik’. statement is given followed by two
‘major go with rifle’ is written as ‘tik arguments. Choose the most
sik lik dik’. appropriate option depending on
123. What is the code for ‘saurabh’? which argument strengthens the
A. mik B. cik given statement.
C. lik D. dik 129.Statement: Should India enact new
E. pik anti-terrorism law?
124.What is the code for ‘army’? Arguments:
A. mik B. cik I. Yes, the existing laws are not
C. lik D. dik strong enough to fight against new
E. pik generation of terrorists.
125.What is the code for ‘major’? II. No, there are laws which are
A. mik B. cik sufficient to keep terrorism under
C. sik D. dik check, but not properly implemented.
E. pik We should ensure proper
126.Whose code is ‘cik’? implementation of the existing laws.
A. convoy B. further A. Only argument I is strong.
C. rifle B. Only argument 2 is strong
D. Either 'further' or 'convoy' C. Either I or II is strong
E. None of these D. Neither I nor II is strong
127.What is the code for ‘go’? E. Both I and II is strong
A. mik B. tik
C. pik Direction: The question given below
D. Either mik or tik consists of a statement, followed by
E. None of these two arguments numbered I and II.
You have to decide which of the
Direction: In the following question arguments is a 'strong' argument and
is given a statement followed by two which is a 'weak' argument.
assumptions numbered I and II. You 130.Statement: Should parents be
have to consider the statement and involved in arranging their child’s
the following assumptions and decide marriage?
which of the assumptions is implicit Arguments:
in the statement. I. Yes, because this is what has been
128.Statement: "You are hereby happening for generations.
appointed as a curriculum developer II. No, because every child should be
with a period of probation that shall given the opportunity to live on their
last for 6 months; at the end of which own terms and to make choices on
you will be confirmed or let go as per their own.
your performance evaluation" - A line A. Only argument I is strong
in an offer letter. B. Only argument II is strong
Assumptions: C. Either I or II is strong
I. The performance of any D. Neither I nor II is strong and
professional can usually not be E. Both I and II are strong
measured at the time of offering the
job. Direction (131 – 135) : Study the
II. The probation period is for the following information carefully and
professional to prove his capabilities. answer the questions that follow.
A. If only assumption I is implicit Twelve people are sitting in two
B. If only assumption II is implicit parallel rows containing six people
C. If either I or II is implicit each such that they are equidistant
D. If neither I nor II is implicit from each other. In row 1, A, B, C, D,

22
E and F are sitting facing south. In decide which of the given conclusions
row 2, P, Q, R, S, T and V are sitting logically follows from the given
facing north. Therefore, in the given statements disregarding commonly
seating arrangement, each member known facts.
sitting in a row faces another 136.Statements:
member of the other row. Three Some calculators are phones.
persons sit between C and D. Either No phone is an eraser.
C or D sits at an extreme end of the Conclusions:
line. The one who faces D sits third to I. No calculator is an eraser.
the left of R. S faces the one who sits II. Some calculators are definitely not
third to the left of A and he cannot sit phones.
adjacent to R. The immediate A. Only conclusion I follow
neighbour of Q faces the immediate B. Only conclusion II follows
neighbour of A. Only one person sits C. Either conclusion I or conclusion II
between P and T, who is facing the follows
one sitting on the immediate right of D. Neither conclusion I nor conclusion
E. Neither E nor F faces R. V and Q II follows
cannot sit adjacent to each other. E. Both conclusion I and conclusion II
131. Who among the following faces B? follow
A. P B. Q
C. T D. R Direction: In the questions below
E. S are given two statements followed by
132.Who among the following sit at the two conclusions numbered I and II.
extreme ends of the rows? You have to take the two given
A. D, Q B. T, F statements to be true even if they
C. E, S D. B, T seem to be at variance from
E. F, Q commonly known facts. Read all the
133.If E is related to P in the same way conclusions and then decide which of
as C is related to S, which of the the given conclusions logically follows
following is A related to, following the from the statements disregarding
same pattern? commonly known facts.
A. V B. R 137.Statements :
C. Q D. T Some plays are shows.
E. Can’t be determined All shows are theatres.
134.How many persons are sitting Conclusions:
between E and F? I. All shows are plays.
A. None B. One II. No play is a theatre
C. Two D. Three A. Only conclusion I follow.
E. Four B. Only conclusion II follows.
135.Four of the following five are alike in C. Either conclusion I or II follows.
a certain way based on the given D. Neither conclusion I nor II follows.
arrangement and so form a group. E. Both conclusions I and II follow.
Which is the one that does not belong
to that group? Direction: In question below are
A. P – D B. Q – A some statements followed by some
C. Q – F D. S – B conclusions. You have to take the
E. T – A given statements to be true even if
they seem to be at variance with
Direction: In question below are commonly known facts and then
some statements followed by some decide which of the given conclusions
conclusions. You have to take the logically follows from the given
given statements to be true even if statements disregarding commonly
they seem to be at variance with known facts.
commonly known facts and then 138.Statements:

23
All computers are machines. number is interchanged with the
All machines are expensive. second last number and so on upto
All expensive are scanners. the fourth number. Which of the
Conclusions: following number is second to the
I. At least some scanners are right of the fourth letter from the left
machines. end?
II. Some computers are not A. 7 B. 4
expensive. C. 6 D. 0
A. Only conclusion I follows. E. None of these.
B. Only conclusion II follows. 142.If we replace all the numbers to its
C. Either conclusion I or conclusion II place letter value. For example, 8 is
follows. replaced by H, then how many
D. Neither conclusion I nor conclusion vowels are there in the series?
II follows. A. Two B. Three
E. Both conclusions I and II follow. C. Four D. Five
E. None of these
Direction: In question below are 143.If we replace all the letters to its
some statements followed by some place number values then how many
conclusions. You have to take the numbers are divisible by 4?
given statements to be true even if A. Seven B. Eight
they seem to be at variance with C. Six D. Five
commonly known facts and then E. None of these
decide which of the given conclusions 144.Which of the following is seventh
logically follows from the given letter to the right of ninth number
statements disregarding commonly from the right end?
known facts. A. Y B. O
139.Statements: C. R D. Q
Some colours are reds. E. I
All reds are blacks. 145.Shalini walked 15m towards south,
No black is a white. took a right turn and walked 3m. She
Conclusions: took a right turn again and walked
I. At least some colours are blacks. 15m before stopping. Which direction
II. All whites are colours. did Shalini face after stopping?
A. Only conclusion I follows. A. West B. South
B. Only conclusion II follows. C. East D. North
C. Either conclusion I or II follows. E. cannot be determined
D. Neither conclusion I nor II follows.
E. Both conclusions I and II follow. Direction (146 – 150) : Study the
following data carefully and answer
Direction (140 – 144) : Study the the questions accordingly.
information given below and answer W, X, Y, T, U, V and Z are seven
the questions based on it. colleagues, who live in seven storey
H*7)0M^%K@L&8P#&OY building. The ground floor is
7I4Q6R@8E6&W8)Z numbered 1; the floor above it is
140.How many letters are there in a numbered 2; and so on, until the
sequence which is immediately topmost floor is numbered 7. Each of
preceded by a number or them works in different companies
immediately followed by a symbol, i.e. IOC, RIL, BPCL, SBI, Tata Motors,
letter counted only once? ONGC and HPCL but not necessarily
A. Six B. Seven in the same order. Z and X don’t work
C. Eight D. Nine in SBI. The one, who works in Tata
E. None of these. motors, lives just above X. The one,
141.If the first number is interchanged who works in HPCL, lives above T. Z
with the last number and the second lives on the fifth floor. Neither X nor

24
Z works in RIL. There are two floors There are three floors between T and
between the floors on which X and V the one, who works in IOC.
lives. The one, who works in ONGC, 146.Which of the following works in IOC?
lives on the topmost floor. V lives on A. The one, who lives just below W
an even numbered floor. Y works in B. The one, who lives on sixth floor
BPCL. There is only one floor between
U and the one, who works in RIL.
C. The one, who lives between V and E. None of these
X 149.Find the odd one.
D. The one, who lives two floors A. W, RIL B. T, BPCL
above X C. Z, Tata motors D. V, SBI
E. None of these E. Y, HPCL
147.If V is related to Tata motors and T is 150.Find the correct statement.
related to HPCL then in the same A. Y works in BPCL and lives just
way, Z is related to - above X
A. RIL B. SBI B. Four persons are living between W
C. BPCL D. ONGC and the one, who works in SBI
E. None of these C. U works in Tata motors and lives
148.How many persons are living above two floors away from T
the one, who works in BPCL? D. V lives three floors away from the
A. Three B. Two one, who works in HPCL
C. Five D. Four E. None is correct

25
ANSWERS
1. Ans. D. 5x2 + 7x – 6 = 0
pattern is ×1+1, ×2+2, ×3+3, ×4+4, 5x2 + 10x - 3x – 6 = 0
×5+5, ×6+6 So x = +3/5, -2
So series is: 3y2 + 19y + 28 = 0
8, 9, 20, (63), 256, 1285, 7716 3y2 + 12y + 7y + 28 = 0
64 is wrong number is the series. So y = -7/3, -4
2. Ans. E. Clearly from values X ˃ Y
pattern is +1 ÷2, +2 ÷2, +3 ÷2, +4 ÷2, 9. Ans. E.
+5 ÷2, +6 ÷2 8x2 - 6x – 5 = 0
(1015+1) ÷2= 508 8x2 + 4x - 10x – 5 = 0
(508+2) ÷2= 255 So x = +5/4, -1/2
(255+3) ÷2= 129 2y2 - 7y – 4 = 0
(129+4) ÷2=66.5 2y2 - 8y + y – 4 = 0
(66.5+5)÷2= 35.75 So y = +4, -1/2
(35.75+6) ÷2= 20.875 X=+5/4 is less than y= +4 and greater
130 is wrong number is the series. than y= -1/2
3. Ans. D. So no relation between x & y.
pattern is ; × 0.5 + 0.5, × 1 + 1, × 1.5 10. Ans. D.
+ 1.5, × 2 + 2, × 2.5 + 2.5, × 3 + 3 6x2 + 5x -4 = 0
So series is: 6x2 + 8x – 3x – 4 = 0
7, 4, 5, 9, 20, (52.5), 160.5 So x = -4/3, +1/2
51 is wrong number is the series. y (10y – 11) = 3
4. Ans. D. 10y2 – 11y +3 = 0
The pattern is :The series is +32, +52, 10y2 – 5y - 6y +3 = 0
+72, +112, +132, .. So y = +1/2, +3/5
So series is 1108, 1117, 1142, 1191, Both value of Y greater than X.
1312, (1481) So X ≤ Y
1480 is wrong number in the series. 11. Ans. A.
5. Ans. D. For Quantity I,
Let assume total unit= 60
Capacity of Tap= 60/15= 4 unit/sec
Capacity of Tap with leakage= 60/20= 3
6. Ans. A. unit/sec
I. x2 - 6x - 7 = 0 Capacity of leak= 4-3= 1unit/sec
x2 - 7x + x - 7 = 0 So time required to empty= 60/1= 60
x (x - 7) + 1 (x - 7) = 0 sec
(x + 1) (x - 7) = 0 For Quantity II, 50 sec
x = -1 or 7
II. 2y2 + 13y + 15 = 0 So Quantity I ˃ Quantity II
2y2 + 10y + 3y + 15 = 0 12. Ans. A.
2y (y + 5) + 3 (y + 5) = 0
(y + 5) (2y + 3) = 0 Quantity I:
y = -5 or -3/2 56x + 126 = 65x + 90
Clearly x > y 9x = 36
7. Ans. B. x=4
2x2 + 17x + 30 = 0 A= 8 × 4 + 18- 13 = 37 years
2x2 + 12x + 5x + 30 = 0 Quantity II: Y2 + 32Y - 185=0
So x = -6, -5/2 Y2 + (37-5)Y - 185=0
4y2 – 7y – 15 = 0 Y= -37, +5
4y2 – 12y + 5y – 15 = 0 So option A , Quantity I ˃ Quantity II
So y = -5/4, 3 13. Ans. B.
So clearly from values Y ˃ X Quantity I: Total CP = 1500

26
Total SP = 1500 + 10% of 1500 = 1500 Required answer = (220 + 340 + 300 +
+ 150 = 1650 450)
CP of 1/3 of bananas = 1500/3 = Rs.500 = 1310
SP of 1/3 of bananas at 25% loss 18. Ans. A.
= 500 – [ (500 x25 / 100)] = 500 - 125
= 375
SP of the rest of bananas = 1650 - 375
= 1275
Now, CP of the test of bananas = 1500 -
500 = 1000
Profit on the rest of bananas = 1275 - The number of customer who buys
1000 = 275 laptops from Jabalpur = 90 + 40% of 90
% of profit on the rest of bananas = = 126
(275/1000)×100 = 27.5% The number of customer who buys
So option B, Quantity II ˃ Quantity I tablets from Jabalpur = 210 – 30% of
14. Ans. C. 210 = 147
Let the distance between A and B be D According to question
km 126 : 147 = 6 : x
Then. [ D/(50/60)] – [D/(56/60)] = 9 ⇒6:7=6:x
Or, 6D/5 – 15D/14 = 9 ⇒x=7
Or, Hence, the required value = x + 3 = 7 +
Distance = 70 km 3 = 10
Quantity II: Y2 – 40Y -2100 = 0 19. Ans. A.
Y2 – (70-30)Y -2100 = 0
Y= +70, -30
So option C, Quantity I ≥ Quantity II
15. Ans. E.
CP = x
SP = x + (x*x)/100 = 56
x2 + 100x – 5600 = 0
x = 40 Required ratio = 450: 60
SP = 40 + (40 * 80)/100 = Rs. 72 = 15: 2
Quantity II: Y2- 95Y+ 1500=0 20. Ans. E.
Y2- (75+20)Y+ 1500=0
Y= +75, +20
So option E, relation can’t determine
between Quantity I & II.
16. Ans. C.

Required average = (99 + 90 + 90) ÷ 3


= 279 ÷ 3
= 93
21. Ans. E.
by using statement I, relative speed of
both train in opposite direction=
Required percentage = × 100 450/12= 37.5 m/sec
= 75% By using statement II, we can find speed
17. Ans. A. of both trains, but can’t determine which
one is faster.
But Both Statement are not sufficient to
determine the length of train B.
22. Ans. C.
by using statement I, in final mixture
milk: water= 27:13

27
Milk= 80×27/40= 54 litres
Water= 80-54= 26 litres
Before adding 8 litres water,
Water was= 26-8= 18 litres
Initial Ratio milk : water= 54:18= 3:1
So initial milk= 80×3/4= 60 litres
Same concept by using statement II,
so both statement are sufficient to Required percentage = × 100
answer alone.
23. Ans. D. = 21.25% 21%
Using Statement I, 27. Ans. A.
Let Rakesh’s present age x years &
Suresh’s present age be y years
x/(y+4) = 5/7
Using alone this statement, we can’t
determine answer.
Using statement II,
(x-4)/y = 2/3 Ratio of number of females to number of
Using alone this statement, we can’t males in train E (all the coaches
determine answer. together) is 11: 24
Using equation (1) and (2) we can solve
for x and y. Number of males in train E = 1680 ×
So, both statements are required to give = 1152
answer. AC passengers in train A and D together
24. Ans. B. = 192 + 288
Statement I = 480
Length:Breadth= 7:3 Required difference = 1152 – 480
Using alone this statement, we can’t = 672
determine answer 28. Ans. C.
Statement II
height = 1/5 of perimeter
For area of all walls= perimeter × height
Cost= area × 750
So statement II alone is sufficient to
answer.
25. Ans. C. Passengers who are not in the AC coach
Using statement I, in train A = 800 – 192
Let ratio of income = 3x:4x = 608
and ratio of expenditure = 4y:5y Passengers who are not in the AC coach
B's saving= 25% of salary= ¼ of salary, in train C = 1920 – 216
So, ATQ, 4x-x = 5y = 1704
x/y = 5/3 Passengers who are not in the AC coach
Ratio between income & expenditure. in train D = 1600 – 288
We can find ratio of savings. = 1312
Using statement II, we can calculate Required average = (608 + 1704 +
income of both and expenditure of both. 1312) ÷ 3
So we can find ratio of savings = 3624 ÷ 3
So either of the statement is sufficient to = 1208
answer. 29. Ans. B.
26. Ans. B.

28
Number of passengers in AC coach in
train B and D together = 240 + 288
= 528
Required ratio = 528: 800
= 33: 50
30. Ans. B.

Required answer = (4200 + 1800) –


(1200 + 1000)
= 6000 – 2200
= 3800
Number of AC passengers in train C and 37. Ans. B.
D together = 216 + 288
= 504

Required percentage = × 100


= 68.5%
31. Ans. B.
Share of Amit : Share of Ruchi : Share
of Roni is Let the cost of Q’s product be 2x and S’s
2000×8 + 2600×4 : 2800×8 + 3200×4 product be 3x.
: 4200×4 A.T.Q,
33 : 44 : 21 2640 × 2x + 2720 × 3x = 1344000
So share of Amit = [33/(33+44+21)]× 5280x + 8160x = 1344000
15680 = Rs 5280 13440x = 1344000
Difficulty Level: moderate x = 100
32. Ans. A. Therefore, revenue of company Q =
Let speed of train= x km/hr 2640 × 2 × 100
{(x-5)×5/18} ×27= 450 = Rs. 528000
X= 65 km/hr 38. Ans. D.
Time to cross platform=
(525+450)/(65×5/18)= 54 Sec
33. Ans. C.
unit convert m into cm
So total required bricks for wall=
(1500×1200×10)/(24×25×10) = 3000
Since 37(1/2)% is alrady built, so
required bricks = (100-37.5)% × 3000
Required answer = 3780 + 800: 2720 +
= 1875
2640
34. Ans. C.
= 4580: 5360
Valid votes = 80% of 7800 = 6240
= 229: 268
1nd candidates got 16(2/3)% more than
39. Ans. C.
other
1/6 more than other
Ratio between one candidate : second
candidates= 7:6
So winner got = 6240×7/(7+6)= 3360
35. Ans. B.
If the required distance be x km, then
x/8 – x/10 = (30-6)/60
x= 16 km Required answer = (3780 + 2640 +
36. Ans. C. 2720 + 1350)/4
= 2622.5
40. Ans. B.

29
According ratio(1), 4500 Rs :
64000+26000
So 3600 Rs :
(64000+26000)×3600/4500= 72000
So in ratio(1), 6A= 72000
A= 72000/6= 12000 Rs.
44. Ans. A.
Total production of company R after Assume total work unit=120
20% increase = 1200 × 1.20 A efficiency= 120/40= 3 unit per day
= 1440 B efficiency= 120/30= 4 unit per day
Total production of company T after 30% Remaining work finished by A in last 5
increase = 1000 × 1.30 days= 3×5= 15 unit
= 1300 Rest work done by both A & B= 120-15=
Sold product of company R = 1440 × 105 unit
0.82 So A & B work for days= 105/(3+4)= 15
= 1180.8 day
Sold product of company T = 1440 × B left after 15 days
0.80 45. Ans. D.
= 1040 Let Mukesh monthly income be Rs. 100
Required sum = 1180.8 + 1040 Then, money spent on household
= 2220.8 expenditure =100×20/100 = Rs. 20
41. Ans. B. ∴ Remaining amount = 100 - 20 = Rs.
amisha, Aaradhana, Alka, Aditi, Aashirya 80
– X, Y, Z, P, Q Money spent on children’s education
X + Y = 35 * 2 =70 –(1) =80×25/100 = Rs. 20
Z + Y = 31 * 2 =62 –(2) Money spent on transport =80×15/100
X + Z = 36 * 2 = 72 –(3) = Rs. 12
From (1) (2) and (3) Money spent on medicine =80×15/100
X = 40 ; y =30; Z = 32 = Rs. 12
Average age of P and Q =1/2 * [( X + Y Money spent on entertainment
+ Z)/3] = 102/6 = 17 =80×10/100 = Rs. 8
Sum of the age of P and Q = 34 ∴ Last remaining amount = 100 - (20 +
Average age of all the five people = (34 20 + 12 + 12 + 8) = 100 - 72 = Rs. 28
+ 102)/5 = 27.2 Now, Rs. 28 is left, when total income is
42. Ans. B. Rs. 100
Let the amount invested in scheme A be Rs. 1 is left, when total income =100/28
Rs.x and that in B be Rs. 3x. ∴ Rs. 9800 is left, when total income
Then, [(x × 4 × 8)/100]+[(3x × 2 × 13) =100/28×9800 = Rs. 35000
/100] = 1320 46. Ans. A.
Or, (32x/100) + (78x/100) = 1320 Let the male students applied for P.O,
110x/100 = 1320 S.O and clerk be x, y and z respectively,
x = (1320 x 100) /110 = Rs. 1200 In addition, female students applied for
43. Ans. B. P.O, S.O and clerk be a, b and c
A invest for 12 month respectively.
B invest after 4 month, invest for 8 Given conditions are
months z = 3x --------- (I)
C invest 6month after B, he invest for 2 y = 120 + x --------- (ii)
months x : a = 2: 3 --------- (iii)
Profit ratio= investment× time b = c --------- (IV)
A×12: 16000×8 : 26000×2 x = 0.75c --------- (v)
6A: 64000: 26000……………….ratio(1) x + y + z + a + b + c = 1220 ---------
Out of total profit 8100 Rs, A get= 3600 (VI)
Rs
So B & C combined Get 8100-
3

30
By putting the values in terms of x from
all the equations in equation (VI), we
get,
x + 120 + x + 3x + 3x/2 + x/0.75 + x/
0.75 = 1220
39x + 16x = 1100 × 6
x = 120
Therefore,
y = 120 + x = 120 + 120 Required ratio = 720: 500 = 36: 25
= 240 48. Ans. B.
z = 3 × 120 = 360 Let the male students applied for P.O,
a = 3x/2 = 180 S.O and clerk be x, y and z respectively,
b = x/0.75 = 160 In addition, female students applied for
b = c = 160 P.O, S.O and clerk be a, b and c
respectively.
Given conditions are
z = 3x --------- (I)
y = 120 + x --------- (ii)
x : a = 2: 3 --------- (iii)
b = c --------- (IV)
x = 0.75c --------- (v)
x + y + z + a + b + c = 1220 ---------
Required answer = 360 + 160= 520 (VI)
47. Ans. C. By putting the values in terms of x from
Let the male students applied for P.O, all the equations in equation (VI), we
S.O and clerk be x, y and z respectively, get,
In addition, female students applied for x + 120 + x + 3x + 3x/2 + x/0.75 + x/
P.O, S.O and clerk be a, b and c 0.75 = 1220
respectively. 39x + 16x = 1100 × 6
Given conditions are x = 120
z = 3x --------- (I) Therefore,
y = 120 + x --------- (ii) y = 120 + x = 120 + 120
x : a = 2: 3 --------- (iii) = 240
b = c --------- (IV) z = 3 × 120 = 360
x = 0.75c --------- (v) a = 3x/2 = 180
x + y + z + a + b + c = 1220 --------- b = x/0.75 = 160
(VI) b = c = 160
By putting the values in terms of x from
all the equations in equation (VI), we
get,
x + 120 + x + 3x + 3x/2 + x/0.75 + x/
0.75 = 1220
39x + 16x = 1100 × 6
x = 120
Therefore,
y = 120 + x = 120 + 120
= 240 Required answer = × 100 =
z = 3 × 120 = 360 25%
a = 3x/2 = 180 49. Ans. D.
b = x/0.75 = 160 Let the male students applied for P.O,
b = c = 160 S.O and clerk be x, y and z respectively,
In addition, female students applied for
P.O, S.O and clerk be a, b and c
respectively.
Given conditions are

31
z = 3x --------- (I) z = 3 × 120 = 360
y = 120 + x --------- (ii) a = 3x/2 = 180
x : a = 2: 3 --------- (iii) b = x/0.75 = 160
b = c --------- (IV) b = c = 160
x = 0.75c --------- (v)
x + y + z + a + b + c = 1220 ---------
(VI)
By putting the values in terms of x from
all the equations in equation (VI), we
get,
x + 120 + x + 3x + 3x/2 + x/0.75 + x/
0.75 = 1220
39x + 16x = 1100 × 6
x = 120 Required answer = × 100 =
Therefore, 50/9%
y = 120 + x = 120 + 120 51. Ans. A.
= 240 The previous sentence talks about
z = 3 × 120 = 360 globalization's opponents drowning their
a = 3x/2 = 180 defenders. The sentence following the
b = x/0.75 = 160 blank also mentions 'Can globalization
b = c = 160 be saved?'. This indicates a possibility of
a downfall. Corresponding to that,
collapse is the most suitable response.
'Collapse' refers to a sudden downfall or
breakdown.
52. Ans. A.
The passage mention every aspect of
globalization – free trade, free
movement of capital, and international
migration – is under attack by forces
Required answer = × 100 = 150% ranging from populist political parties to
50. Ans. D. separatist groups to terrorist
Let the male students applied for P.O, organizations. This indicates that the
S.O and clerk be x, y and z respectively, forces are of hostile nature which is best
In addition, female students applied for implied by the use of the word
P.O, S.O and clerk be a, b and c 'antagonistic'. It refers to showing or
respectively. feeling active opposition or hostility
Given conditions are towards someone or something.
z = 3x --------- (I) Couthy means warm and friendly nature
y = 120 + x --------- (ii) of a person.
x : a = 2: 3 --------- (iii) Chummy means on friendly terms;
b = c --------- (IV) friendly.
x = 0.75c --------- (v) Pally means having a close, friendly
x + y + z + a + b + c = 1220 --------- relationship.
(VI) Providential means occurring at a
By putting the values in terms of x from favourable time; opportune.
all the equations in equation (VI), we 53. Ans. E.
get, 'Condemn' means to express complete
x + 120 + x + 3x + 3x/2 + x/0.75 + x/ disapproval; censure. It is the most
0.75 = 1220 suitable response according to the
39x + 16x = 1100 × 6 context of the statement.
x = 120 Complimentary means expressing a
Therefore, compliment; praising or approving.
y = 120 + x = 120 + 120 Flattering means full of praise and

32
Absolving means declare (someone) free particular part of the population as a vote
from guilt, obligation, or punishment. bank without thinking about other Indian
Laudatory means (of speech or writing) citizens.
expressing praise and commendation. B and D can be connected since
54. Ans. B. contextually it states that claims and
'Interference' refers to the action of objectives of providing shelter to the
interfering(intervening) or the process of minorities in India’s neighbourhood in the
being interfered(intervened) with. The bill are less in reality than what
passage is talking about anti-Western government claimed in the bill. So
groups campaigning against contextually a complete meaning can be
globalization, aversion to European inferred. As far as grammar is concerned,
integration by populist parties in Europe “claims and objectives” are the nouns
and immigration. Corresponding to the which further need a verb or a preposition
context of the statement, 'Interference' is to complete a meaningful sentence thus,
the most suitable response (in relation to “F and E” are not fit to be added after
foreign activity). “claims and objectives”. So, the correct
Compaction means the process by which pair is B and D. C and E cannot be
the porosity of a given form of sediment connected as we may infer something
is decreased as a result of its mineral contextually but grammatically it will be
grains being squeezed together by the incorrect as the first or main clause is in
weight of overlying sediment or by past tense thus the subordinate clause of
mechanical means. the sentence cannot be in present and
Facilitation means the act of helping other should be in past. C and E therefore,
people to deal with a process or reach an cannot make a correct sentence.
agreement or solution Thus, option E is the correct answer.
Compunction means a feeling of guilt or 57. Ans. C.
moral scruple that prevents or follows the C and D can be joined to form a
doing of something bad. meaningful as well as a grammatically
55. Ans. D. correct sentence. Sentence “C” states a
Among the given options, only fact and the effect of it has been shown
organisation and institutions seem to fit with the help of sentence D.
in. However, 'organisation' is singular A and E cannot be connected as the
while the sentence talks about plural context of both the sentences are
entities. Hence, the most apt response is different from each other. In Sentence
'institutions'. “A”, “congress” refers to a convention or
Assimilation means the process of taking conference is talked about and in
in and fully understanding information or sentence E, a political party is being
ideas. talked about and the context of the
Distraction means a thing that prevents sentences is also not aligning to each
someone from concentrating on other. B and F cannot be connected as
something else well as the subject in part F is 'hatred'
56. Ans. E. (singular) for which the verb used is 'are'
A and F can be connected as it conveys (plural). This violates sentence verb
the meaning both grammatically and agreement rule and is thus erroneous.
contextually as well. Sentence “A” ends Thus, option C is the correct answer.
with “over” which is a preposition and 58. Ans. B.
thus it needs an object or an objective In order to form a meaningful and
clause after it so, “D” and “E” cannot grammatically correct sentence, the two
come after “over” as both of them start parts of a sentence should connect
with a preposition. Hence, “F” will fit grammatically and contextually. In the
which makes the sentence grammatically given question, A joins with D to make a
correct. Contextually as well, we can infer correct sentence. B tells about a word
that sentence intends to mention the aim that could be used to describe Christmas
of someone which is to aim and make a The word has been mentioned in F,

33
making B-F a pair. Contextually, C and E All the highlighted words are in correct
could have been joined, if the conjunction position and no exchange is required.
“and” had been placed between the verbs Hence, the correct answer is E.
“detect and distinguish”. Thus, option B is 67. Ans. A.
the correct answer. Partition of India was one particular
59. Ans. C. happening in the history and thus be
Option A mentions a person, who can preceded by singular noun, so the word
have a certain behaviour, thus, it can pair at A is incorrect. The noun following the
up with E. Option C must go with D as it phrase "one of the" is always a plural
explains that lack of fitness is the reason noun, so the word at D is incorrect.
why most players don’t play for long. Further, it was not the migration that led
Only these two make sense. The correct to "tragic death and displacement".
answer is option C. People were displaced in large numbers;
60. Ans. C. hence, it was called migration. The
Option A must go with E as it talks about Partition of India on the other hand was
a player who was very good in the early an event, which had many other aspects
days but doesn’t play as well these days. other than just migration. So, A & D
Option C must go with D as it talks about should be exchanged. Hence, the correct
an incident when a cat slipped over a answer is A.
pencil that was lying on the floor. Rest of 68. Ans. A.
the option doesn’t make sense. Thus, the ‘Prepare’, ‘engineering’ and
correct answer is option C. 'consequences' are placed correctly in the
61. Ans. C. sentence and need not be interchanged.
Refer to the last sentence in the series. ‘Cope with’ is used when some difficult
62. Ans. E. situation is given. Also, ‘cope (up) with’ is
Refer to the last sentence in the series. a phrase with which we use a noun.
63. Ans. A. Hence, ‘risk’ should be interchanged with
Refer to the last sentence in the series. ‘better’ in order to make the statement
64. Ans. D. contextually as well as grammatically
Refer to the last sentence in the series. correct.
65. Ans. B. 69. Ans. D.
The paragraph is talking about decrease The highlighted words have been
in number of lions in the Gir Forest. The correctly placed in the sentence. Thus,
sentence that introduces this topic is option D is the correct answer.
option C. So, C comes first. E could also 70. Ans. E.
work as an introduction. But, E introduces The statement explains that if the current
the topic of 'infighting in the wild' which situation of economic growth continuous,
doesn’t match with other sentences in the one million species could go extinct in
paragraph. Now, for the next sentence, decades. So, after interchanging
we need elaboration on the topic ‘factoring’ with ‘continuous’ and ‘decades’
introduced in C. F does this perfectly as it with ‘extinct’, we get a meaningful
gives statistics about the death of lions. statement. Hence, option E is the right
Now, an event took place, so we would answer.
have consequences. An immediate 71. Ans. C.
consequence of the events is that an “To contend with” means to have to deal
inquiry was made, as described in A. So, with a difficult or unpleasant situation.
A comes next. So far, our sequence is Millennials (a word crafted to refer to the
CFA. The next is D as it is fixed. The generation born between 1980 and 2000)
remaining two sentences are B & E. E are young people who are between the
should come after D as both have a ages of 20 to 40 years and are fast
common topic of 'infighting'. This leaves entering their prime spending years.
for the last. So, the sequence becomes Growing up in an age of rapid change,
CFADEB. they have a set of expectations and
66. Ans. E. priorities that are radically different from

34
those of the older generations. assumption.
The passage talks about the importance All the statements can be taken as an
of saving for the future or retirement. assumption made by the author in order
Saving for the future is on the minds of to support his/her argument, except
millennials, but many feel overwhelmed statement B. The author does not talk
and under-prepared. Coupled with rising about ‘small amount’ or ‘larger amount.’
health care, child care, and housing costs, He/she is trying to say that if you’re 20
the economic priority for millennials is to today, and put $1 aside, and it earns the
achieve and maintain financial stability historical 6.6% return, by the time you’re
instead of saving for the future. They’re 65, that single dollar will have become
regularly chastised for failing to save $18.50. But if you put aside the same
enough for their future. dollar at 30, by 65 it’s only worth $9.60 –
Millennial paradox refers to the situation you’ve lost half of the potential gains.
where they know “they should be saving Therefore, option B is the apt answer.
£800 (or about $1,146) a month over the 74. Ans. A.
next 40 years, in order to retire at 65 with The fourth paragraph says that the cost
an annual income of £30,000” but of living is expensive and it’s hard to save
“they’re too busy buying groceries or for the future. Millennials still have to
paying rent” and debts “to even think work hard to continue to support
about being able to have that much themselves and pay for the necessities
money to allocate to a savings account.” and recreational needs they have. The
Basically, “the problem is that there are lack of saving culture means they are
too many other factors stopping spending more on current consumption
millennials from making that decision to instead of saving for future consumption.
save.” Less accumulation of wealth means less
This idea is best described in statement security on their future consumption,
C. Therefore, option C is the apt answer. which will result in uneven consumption
72. Ans. E. patterns in their life. Thus, this inference
Abate is a verb which refers to something properly follows from the statements of
unpleasant or severe that become less facts given in the fourth paragraph.
intense or widespread. In other words, it Therefore, option A is the apt answer.
means to make something less intense. 75. Ans. B.
Option I: Wane refers to a state or feeling The fourth paragraph says that the most
which decrease in vigour or extent; important parts of the economy, like
become weaker. housing, healthcare, education, are
Option II: Recede refers to a quality, getting more expensive. Consequently, it
feeling, or a possibility that gradually creates an even bigger financial burden
diminish. for millennials to shoulder. Facing a stark
Option III: Prolong means to extend the set of financial circumstances, millennials
duration of something. started adulthood with less room for
‘Wane’ and ‘recede’ are synonyms of financial mistakes than previous
abate and ‘prolong’ is antonym of abate. generations. In response, they are
Therefore, option E is the apt answer. managing their money differently.
73. Ans. B. This situation can either be an excuse for
An argument is an expression of opinion hopelessness, apathy, and retreat, or an
that is developed in a passage. Some opportunity to rise to the occasion, to find
arguments contain hidden assumptions a better path, to zig when the world zags
sometimes intended to subtly bias the — to make the obstacle the way. Many
reader. An assumption is a belief that the millennials are refinancing student loans,
author takes for granted, based on delaying home purchases, and looking for
opinion or experience. Assumptions are creative ways to earn more money
presented as facts. If the author states an through side hustles. They are taking the
opinion but doesn’t supply any supporting latter course. By making these
details, you should suspect that it is an countermoves around the obstacles in the

35
economic landscape, millennials can Mundane means
become more financially savvy, clever or i) characterized by the practical,
shrewd. transitory, and ordinary
Note that it is based on the idea that ii) of, relating to, or characteristic of the
many millennials, not all millennials, are world; earthly
becoming financially savvy. One can say In the first statement, mundane as in
that it is based on probability and not true ordinary affairs/situations fits in while in
for all. Thereby, this inference is not the second statement, mundane as in
definitely true in light of the facts given in earthly fits in.
the fourth paragraph. Exultant means happy which, doesn't fit
Therefore, option B is the apt answer. in the second sentence.
76. Ans. D. Desolate as a noun means barren and as
The error lies in option D. The correct a verb means to ruin.
idiom is ‘huff and puff’ which means Hence, mundane is the correct answer.
disapproving to complain loudly and 80. Ans. A.
express disapproval. Therefore, option D The meanings of the words are as follows:
is the apt answer Solace: comfort or consolation in time of
77. Ans. A. great distress or sadness.
Option A: Sanguine refers to a piece of Forsaken: abandoned.
writing that is optimistic or positive, Languid: weak or faint from illness or
especially in an apparently bad or difficult fatigue.
situation. Gloomy: dark or poorly lit.
Option B: Constructive refers to a piece Robust: strong and healthy .
of writing that has or intended to have a If we pay attention to the contexts of both
useful or beneficial purpose. the sentences, we can easily infer a
Option C: Sarcastic refers to a piece of positive tone, and 'solace' and 'Robust'
writing that is marked by the irony in are the only alternatives which makes a
order to mock or convey contempt. positive sense out of the given ones,
Option D: Satirical refers to a piece of contextually as well as grammatically.
writing in which vices, follies, abuses, and Further we need a noun in the blanks, so
shortcomings are held up to ridicule, we can dismiss 'robust' as it is an
ideally with the intent of shaming adjective.
individuals, corporations, government, or So, the correct answer is option A.
society itself into improvement. 81. Ans. D.
Option E: Inspiring refers to a piece of The meaning of the words are as follows:
writing that has the effect of inspiring Rescind: cancel
someone. Regression: the act of returning to a prior
“Perhaps there is some creative way to state
tackle this” shows that the author is Restitution: the restoring of something
positive regarding the whole situation. that is stolen or lost to the property owner
Among all the options, ‘sanguine’ is the Rendezvous: an arranged meeting that is
most suitable. Therefore, option A is the often held in secrecy
apt answer. Welter: a confused multitude of things
78. Ans. C. Thus, option D fits in both the sentences.
Benefit- an advantage or profit gained 82. Ans. C.
from something. The first sentence talks about a protest,
Censure- express severe disapproval of which means there was unrest. The word
(someone or something), especially in a in the blank must be closer in meaning to
formal statement. the same.
Respite- a short period of rest or relief ‘Rampant’ would mean ‘uncontrolled and
from something difficult or unpleasant. widespread’ in the context of the first
statement and ‘upright or erect’ in the
Only "respite" can fit in both the blanks. second. So, option C is correct.
79. Ans. D. 83. Ans. C.

36
The error in the third part is the use of lucrative window to make job creation the
preposition “of” after the word “reaction”. heart of policy formulation.
In the given sentence, the "reaction" is 89. Ans. D.
related to a situation, not a person. So, The first sentence asserts that there are
“to” should be used here. Hence, option C awards exclusively meant for the
is correct. sportsmen. The second sentence, in this
84. Ans. C. reference, proposes that as a
The error in the third part is that a consequence of the first sentence, the
preposition should be used after the sportspersons should not be considered
words “tempt them” and “to” is the for Padma Awards.
perfect preposition suited to be used here
as it implies approaching something. Thus, both "when" and "as" can be put in
Hence option C is correct. the beginning to join the two sentences.
85. Ans. A. 90. Ans. C.
The preposition ‘across’ is used for First sentence states a fact. It tells us that
depicting something that happened from in India, organised sector generates a
one side to the other side of a place, small number of jobs. Second part states
which in this case is the USA. ‘Into’, on cause of the above fact. Hence ‘Few
the other hand, shows the movement of organised sector jobs get generated
something inside of something else. because’ is the correct option.
Clearly, the correct answer is option A. New Sentence: Few organised sector
86. Ans. A. jobs get generated in India because
The error in the first part is the use of industries prefer capital-intensive
“would” because the sentence is a production despite the economy’s relative
general statement and would use the abundance of low-wage labour.
simple present tense. Thus, “Will” should So, the correct answer is option C.
be used in place of “would”. Hence, option 91. Ans. B.
A is correct. Second sentence states a fact. It tells us
87. Ans. A. that officials of the forest department are
The error in the first part is the use of not given any special training about
“remembers” because the sentence is in medicinal plants. First part is effect of the
past tense and using passive voice, hence above fact. Hence ‘As the officials of the
it should be using past tense of forest department’ is the correct option.
“remember” i.e. “remembered”. Hence, New Sentence: As the officials of the
option A is correct. forest department are not given any
88. Ans. B. special training about medicinal plants,
It can be gathered from the given they are not even able to identify these
sentence that the two things that India plant species.
has gained, have occurred as a So, the correct answer is option B.
consequence of the announcement of the 92. Ans. D.
periodic labour force surveys. Hence, the First sentence states a fact. It states that
phrase "with the announcement" can be Nehru (First Prime Minister of India) is not
used to begin the sentence. Also, the known as decisive leader In India. First
correlative conjunctions "not only... but sentence is not in conformity to second
also" can effectively join the two sentence which state that there is enough
sentences. evidence of Nehru being decisive leader.
Hence both the options ‘Despite
New statement: overwhelming evidence’ & ‘Though there
With the announcement of periodic labour is overwhelming evidence’ are correct.
force surveys, India has not only created Hence Option D is the right answer.
an opportunity for itself to undo its recent New Sentence(s):
lukewarm performance on the job Despite there being overwhelming
creation front but has also opened a evidence of his being a decisive leader,

37
Nehru is hardly ever acknowledged as older generation. And as a consequence,
one. the number of the old and the retirees has
Though there is overwhelming evidence increased in the non- metropolitan
of Nehru being a decisive leader, he is regions.
hardly ever acknowledged as one. 98. Ans. B.
93. Ans. C. The concerned paragraph of the passage
The passage talks about biotechnology talks about the requirements to
being the hope of America’s future. It also contribute to a sound rural biotechnology.
emphasises that the technology can best The first sentence talks about strategic
thrive in rural America, where strategic partnerships between business and
investments and sound partnerships can academia. So, to express the relation
be extremely helpful. These ideas are between community colleges and local
best covered in option C., which is the businesses in the given context, we can
correct answer. use the term “partnership”. The union can
94. Ans. E. also lead to the achievement mentioned
Refer to the ninth and tenth line of the in the same sentence: “to provide a
passage. It talks only about alternatives pipeline of individuals with the skills…”
B and C. Alternative A is not mentioned in Now, since two entities have been talked
the passage. Thus, option E is the correct about, we will use “between” instead of
answer. “among”. Thus, option B is the correct
95. Ans. B. answer.
To wipe- out means to destroy or erase. 99. Ans. B.
The given context implies that technology To be stuck “in a bind” means to be
has been responsible for eliminating the caught in a difficult or awkward situation,
traditional job. Thus, option B is the especially one that is not easy to resolve
correct answer. or escape. Thus, option B is the correct
96. Ans. E. answer.
All the statements are correct with 100. Ans. C.
reference to passage. Let us refer to the The concerned sentence in the passage
following excerpts from the passage: talks about boosting various types of
a. “Under the Bio-Belt initiative, partnerships between businesses and
innovation would grow in rural areas, and academia. Thus, “promote” can replace
biomanufacturing could expand “incentivise”.
across the country, where land and 101. Ans. A.
feedstock are abundant.” Persons: F, G, H, I, J, K, P and Q
b. “Fermentation is an increasingly Outside: 5, Inside: 3
powerful force for converting sugar and Steps:
other forms of biomass into value- 1) F sits second to the right of H.
added goods…” 2) G is not an immediate neighbour of
c. “Agriculture extension schools, which both H and F.
are often located in rural areas, should 3) Neither H nor F sits opposite to G.
expand to deliver research and job
training for rural communities.”
d. “A critical component of our economic
growth will be the bioeconomy.” This
implies that the author of the passage
believes that bioeconomy would be one of
the major contributors to economic
growth of America.
4) Q sits third to the right of F.
97. Ans. C.
5) K sits second to the right of Q.
The given sentence means that the
6) I does not sit next to G.
metropolitan areas have attracted the
younger population, while the non-
metropolitan areas have attracted the

38
7) P sits third to the right of I.
Clearly, 4 persons sit between Q and J,
8) J and G face the same direction as I
counted from the right of Q.
faces.
103. Ans. C.
9) Among them only three are facing the
Persons: F, G, H, I, J, K, P and Q
centre of the table. It means case II is
Outside: 5, Inside: 3
ruled out.
Steps:
1) F sits second to the right of H.
2) G is not an immediate neighbour of
both H and F.
3) Neither H nor F sits opposite to G.

Clearly, K sits second to the left of I.


102. Ans. D.
Persons: F, G, H, I, J, K, P and Q
Outside: 5, Inside: 3
Steps:
4) Q sits third to the right of F.
1) F sits second to the right of H.
5) K sits second to the right of Q.
2) G is not an immediate neighbour of
6) I does not sit next to G.
both H and F.
3) Neither H nor F sits opposite to G.

7) P sits third to the right of I.


4) Q sits third to the right of F. 8) J and G face the same direction as I
5) K sits second to the right of Q. faces.
6) I does not sit next to G. 9) Among them only three are facing the
centre of the table. It means case II is
ruled out.

7) P sits third to the right of I.


8) J and G face the same direction as I
faces.
9) Among them only three are facing the Clearly, P sits opposite to F.
centre of the table. It means case II is 104. Ans. C.
ruled out. Persons: F, G, H, I, J, K, P and Q
Outside: 5, Inside: 3
Steps:

39
1) F sits second to the right of H.
2) G is not an immediate neighbour of
both H and F.
3) Neither H nor F sits opposite to G.

7) P sits third to the right of I.


8) J and G face the same direction as I
faces.
9) Among them only three are facing the
4) Q sits third to the right of F. centre of the table. It means case II is
5) K sits second to the right of Q. ruled out.
6) I does not sit next to G.

7) P sits third to the right of I.


Clearly, Q sits immediate left of P.
8) J and G face the same direction as I
106. Ans. D.
faces.
9) Among them only three are facing the
centre of the table. It means case II is
ruled out.

Going through the options we can say


that D is the only option.
107. Ans. E.
As R can be Niece or Nephew to Q. the
answer cannot be determined.
Clearly, only K faces the centre. 108. Ans. C.
105. Ans. B. T≥S=P≥Q
Persons: F, G, H, I, J, K, P and Q I. P = Q (false)
Outside: 5, Inside: 3 II. P > Q (false)
Steps: But both will follow together
1) F sits second to the right of H. Hence, either conclusion I nor II is true.
2) G is not an immediate neighbour of 109. Ans. D.
both H and F. S < T, T > R, T = W
3) Neither H nor F sits opposite to G. R<T≥S
I. R < S (false)
R<T=W≥S
II. S > W (false)
Hence, neither conclusion I nor II is
true.
110. Ans. C.
X=Y<Z>W
4) Q sits third to the right of F. X<Z
5) K sits second to the right of Q. I. Z = X
6) I does not sit next to G.

40
II. Z > X
Hence, either conclusion I or II is true.
111. Ans. A.
R < S can be clearly deduced from the
relationship.
C ≥ E cannot be true as both are related
by opposite signs.
Thus only conclusion I is true.
Hence Option A is correct Step 3: The head of the class on Monday
112. Ans. B. is neither Student E nor F. So table 2
V < U cannot be true because both are possibility is cancelled. And now we shall
related by opposite signs. proceed with table 1 possibility.
L ≥ S clearly holds true. Step 4: The difference between the roll
Thus only conclusion II follows. number of the head of the class on the
Hence Option B is correct first day of the week and last day of the
113. Ans. A. week is 1.
The first is logical as the statement talks
about the setting up of infrastructure
which is ineffective in providing power to
many areas. The second does not
provide a solution as to why it will help
overcome the power problem.
114. Ans. A.
Only I follows: it speaks about
knowledge and skill development. IQ
and General Knowledge are NOT
synonymous. Hence II doesn't follow. Step 5: The difference of roll number of
115. Ans. C. student B and student F is 4. For this to
The highest roll number belongs to the be true, the student B must have the roll
student who is the head of the class on number 17 and F shall be the head of
Wednesday. the class on Friday with his roll number
Step 1: The head of the class on as 21 as shown in the table below.
Wednesday is Student D and on
Saturday is B. The student whose roll
number is 20 becomes the head of the
class on Thursday.

Finally filling the remaining places, we


get that Student E becomes the head of
the class on Thursday and the student C
who is the head of the class on Tuesday
has 23 roll number.

Step 2: Student C becomes the head of


the class just after student A. The
student with roll number 24 is the next
student to become the head of the class
after student C. Here we can have two
possibilities,

41
116. Ans. A. student B and student F is 4. For this to
The highest roll number belongs to the be true, the student B must have the roll
student who is the head of the class on number 17 and F shall be the head of
Wednesday. the class on Friday with his roll number
Step 1: The head of the class on as 21 as shown in the table below.
Wednesday is Student D and on
Saturday is B. The student whose roll
number is 20 becomes the head of the
class on Thursday.

Finally filling the remaining places, we


get that Student E becomes the head of
the class on Thursday and the student C
who is the head of the class on Tuesday
Step 2: Student C becomes the head of has 23 roll number.
the class just after student A. The
student with roll number 24 is the next
student to become the head of the class
after student C. Here we can have two
possibilities,

Student E
117. Ans. D.
The highest roll number belongs to the
student who is the head of the class on
Wednesday.
Step 1: The head of the class on
Step 3: The head of the class on Monday Wednesday is Student D and on
is neither Student E nor F. So table 2 Saturday is B. The student whose roll
possibility is cancelled. And now we shall number is 20 becomes the head of the
proceed with table 1 possibility. class on Thursday.
Step 4: The difference between the roll
number of the head of the class on the
first day of the week and last day of the
week is 1.

Step 2: Student C becomes the head of


the class just after student A. The
student with roll number 24 is the next
student to become the head of the class
Step 5: The difference of roll number of after student C. Here we can have two

42
possibilities,

38
118. Ans. B.
Step 3: The head of the class on Monday The highest roll number belongs to the
is neither Student E nor F. So table 2 student who is the head of the class on
possibility is cancelled. And now we shall Wednesday.
proceed with table 1 possibility. Step 1: The head of the class on
Step 4: The difference between the roll Wednesday is Student D and on
number of the head of the class on the Saturday is B. The student whose roll
first day of the week and last day of the number is 20 becomes the head of the
week is 1. class on Thursday.

Step 2: Student C becomes the head of


the class just after student A. The
Step 5: The difference of roll number of student with roll number 24 is the next
student B and student F is 4. For this to student to become the head of the class
be true, the student B must have the roll after student C. Here we can have two
number 17 and F shall be the head of possibilities,
the class on Friday with his roll number
as 21 as shown in the table below.

Step 3: The head of the class on Monday


is neither Student E nor F. So table 2
possibility is cancelled. And now we shall
proceed with table 1 possibility.
Step 4: The difference between the roll
Finally filling the remaining places, we
number of the head of the class on the
get that Student E becomes the head of
first day of the week and last day of the
the class on Thursday and the student C
week is 1.
who is the head of the class on Tuesday
has 23 roll number.

43
Step 5: The difference of roll number of Step 2: Student C becomes the head of
student B and student F is 4. For this to the class just after student A. The
be true, the student B must have the roll student with roll number 24 is the next
number 17 and F shall be the head of student to become the head of the class
the class on Friday with his roll number after student C. Here we can have two
as 21 as shown in the table below. possibilities,

Step 3: The head of the class on Monday


is neither Student E nor F. So table 2
possibility is cancelled. And now we shall
Finally filling the remaining places, we
proceed with table 1 possibility.
get that Student E becomes the head of
Step 4: The difference between the roll
the class on Thursday and the student C
number of the head of the class on the
who is the head of the class on Tuesday
first day of the week and last day of the
has 23 roll number.
week is 1.

23
119. Ans. D.
Step 5: The difference of roll number of
The highest roll number belongs to the
student B and student F is 4. For this to
student who is the head of the class on
be true, the student B must have the roll
Wednesday.
number 17 and F shall be the head of
Step 1: The head of the class on
the class on Friday with his roll number
Wednesday is Student D and on
as 21 as shown in the table below.
Saturday is B. The student whose roll
number is 20 becomes the head of the
class on Thursday.

44
Here we have to match the words in all
statements such as ‘go’ come in every
statement so assign it any diagram or
Finally filling the remaining places, we you can write down it and as par word
get that Student E becomes the head of we have to search code in coded
the class on Thursday and the student C statements such as here only ‘tik’ comes
who is the head of the class on Tuesday for three times so it would be the code
has 23 roll number. for ’go’.
Hence ‘miik’ is the code for ‘army’.
125. Ans. C.

Here we have to match the words in all


120. Ans. C. statements such as ‘go’ come in every
statement so assign it any diagram or
you can write down it and as par word
we have to search code in coded
Sia takes 40 min statements such as here only ‘tik’ comes
121. Ans. D. for three times so it would be the code
for ’go’.
Hence ‘sik’ is the code for ‘major’.
126. Ans. D.
Palak takes maximum time
122. Ans. A.

Four persons
123. Ans. E.

Here we have to match the words in all


statements such as ‘go’ come in every
statement so assign it any diagram or
you can write down it and as par word
we have to search code in coded
Here we have to match the words in all statements such as here only ‘tik’ comes
statements such as ‘go’ come in every for three times so it would be the code
statement so assign it any diagram or for ’go’.
you can write down it and as par word Here we can’t match the code for convoy
we have to search code in coded and further but in statement 2 code gik
statements such as here only ‘tik’ comes and cik are remaining, but we can’t
for three times so it would be the code assure that which code will be for which
for ’go’. word. So here we use either or.
Hence ‘pik’ is the code for ‘saurabh’. Hence ‘cik’ is the code for either convoy
124. Ans. A. or further.

45
127. Ans. B. 2. S faces the one who sits third to the
left of A and he cannot sit adjacent to R.
Therefore CASE II is not true.

Here we have to match the words in all


statements such as ‘go’ come in every
statement so assign it any diagram or 3. The immediate neighbour of Q faces
you can write down it and as par word the immediate neighbour of A. Only one
we have to search code in coded person sits between P and T, who is
statements such as here only ‘tik’ comes facing the one sitting on the immediate
for three times so it would be the code right of E. Neither E nor F faces R. V and
for ’go’. Q cannot sit adjacent to each other.
Hence ‘tik’ is the code for ‘go’.
128. Ans. E.
The professional who has just been
recruited needs to be evaluated over a
period of time to know if he/she suits
the work environment of the company.
So, I is implicit. The statement mentions
that the individual's capabilities shall be 132. Ans. B.
judged before confirmation. Hence, II is 1. Three persons sit between C and D.
implicit as well. Either C or D sits at an extreme end of
129. Ans. C. the line. The one who faces D sits third
Since, it is stated that the existing laws to the left of R.
are not strong enough, I is strong
Argument II states that there is no
problem with the laws, but the problem
is with implementation. Hence, II is
strong. But, I and II are contradictory.
One argument says that they are not
sufficient and the other says they are 2. S faces the one who sits third to the
sufficient. Hence either I or II is strong. left of A and he cannot sit adjacent to R.
130. Ans. B. Therefore CASE II is not true.
Argument I is weak – just because
something has been taking
place/happening for ages doesn’t make
that thing right or correct. Argument II
is strong as it gives us a proper
reasoning why parents shouldn’t be
3. The immediate neighbour of Q faces
involved in arranging their child’s
the immediate neighbour of A. Only one
marriage. Therefore, option B is the
person sits between P and T, who is
correct answer.
facing the one sitting on the immediate
131. Ans. D.
right of E. Neither E nor F faces R. V and
1. Three persons sit between C and D.
Q cannot sit adjacent to each other.
Either C or D sits at an extreme end of
the line. The one who faces D sits third
to the left of R.

46
133. Ans. B. 3. The immediate neighbour of Q faces
1. Three persons sit between C and D. the immediate neighbour of A. Only one
Either C or D sits at an extreme end of person sits between P and T, who is
the line. The one who faces D sits third facing the one sitting on the immediate
to the left of R. right of E. Neither E nor F faces R. V and
Q cannot sit adjacent to each other.

2. S faces the one who sits third to the


135. Ans. C.
left of A and he cannot sit adjacent to R.
1. Three persons sit between C and D.
Therefore CASE II is not true.
Either C or D sits at an extreme end of
the line. The one who faces D sits third
to the left of R.

3. The immediate neighbour of Q faces


the immediate neighbour of A. Only one
person sits between P and T, who is
facing the one sitting on the immediate
right of E. Neither E nor F faces R. V and 2. S faces the one who sits third to the
Q cannot sit adjacent to each other. left of A and he cannot sit adjacent to R.
Therefore CASE II is not true.

134. Ans. D.
1. Three persons sit between C and D. 3. The immediate neighbour of Q faces
Either C or D sits at an extreme end of the immediate neighbour of A. Only one
the line. The one who faces D sits third person sits between P and T, who is
to the left of R. facing the one sitting on the immediate
right of E. Neither E nor F faces R. V and
Q cannot sit adjacent to each other.

2. S faces the one who sits third to the


left of A and he cannot sit adjacent to R. 136. Ans. D.
Therefore CASE II is not true.

Neither conclusion I nor II follows.

47
137. Ans. D. 12 & 8 16 # & 15 25 7 9 4 17 6 18 @ 8
The Venn diagram for the above relation 5 6 & 23 8 ) 26
is Numbers divisible by 4 :
8,12,8,16,4,8,8.
144. Ans. E.
Case 1: H * 7 ) 0 M ^ % K @ L & 8 P #
&OY7I4Q6R@8E6&W8)Z
In question, first see ninth number
(i.e. '1','2','3' etc.) from the right end
138. Ans. A. -7
Then, seventh letter (i.e. letter
means 'a','b','c' etc) to the right of
'7' - 'I'
145. Ans. D.
Let Shalini started at A and moved into
south 15 m and reached at B, then she
turned right and walked 3m to reach C.
She again turn right and walked 15m
and reached D.

Only conclusion I follows.


139. Ans. A.
The Venn Diagram for the above relation
is

From the figure it is clear that at point D


Shalini is faced on north direction.
146. Ans. D.
1) Z lives on the fifth floor.
2) The one, who works in ONGC, lives on
the topmost floor.
Thus only conclusion I follow from the 3) The one, who works in Tata motors,
above diagram. lives just above X.
Hence Option A is correct 4) V lives on an even numbered floor.
140. Ans. D. 5) There are two floors between the
Case 1: H * 7 ) 0 M ^ % K @ L & 8 P # floors on which X and V lives.
&OY7I4Q6R@8E6&W8)Z
141. Ans. C.
Case 1: H * 7 ) 0 M ^ % K @ L & 8 P #
&OY7I4Q6R@8E6&W8)Z
Case 2: H * 8 ) 6 M ^ % K @ L & 8 P #
&OY6I4Q7R@8E0&W7)Z
Here, L is the fourth letter and as per
our problem, second to the right of L is
6.
142. Ans. B.
Case 1: H * 7 ) 0 M ^ % K @ L & 8 P #
&OY7I4Q6R@8E6&W8)Z
Case 2: H * G ) 0 M ^ % K @ L & H P #
&OYGIDQFR@HEF&WH)Z
143. Ans. A.
Given : H * 7 ) 0 M ^ % K @ L & 8 P #
&OY7I4Q6R@8E6&W8)Z
After changes : 8 * 7 ) 0 13 ^ % 11 @

48
Therefore, option D is the correct
answer.
6) Neither X nor Z works in RIL. 147. Ans. D.
7) The one, who works in HPCL, lives 1) Z lives on the fifth floor.
above T. 2) The one, who works in ONGC, lives on
8) There is only one floor between U and the topmost floor.
the one, who works in RIL. 3) The one, who works in Tata motors,
9) There are three floors between T and lives just above X.
the one, who works in IOC. 4) V lives on an even numbered floor.
10) Y works in BPCL. 5) There are two floors between the
floors on which X and V lives.

11) Z and X don’t work in SBI.


(Here, case 2 will be eliminated)
6) Neither X nor Z works in RIL.
7) The one, who works in HPCL, lives
above T.

49
8) There is only one floor between U and 4) V lives on an even numbered floor.
the one, who works in RIL. 5) There are two floors between the
9) There are three floors between T and floors on which X and V lives.
the one, who works in IOC.
10) Y works in BPCL.

6) Neither X nor Z works in RIL.


7) The one, who works in HPCL, lives
above T.
8) There is only one floor between U and
the one, who works in RIL.
9) There are three floors between T and
the one, who works in IOC.
11) Z and X don’t work in SBI.
10) Y works in BPCL.
(Here, case 2 will be eliminated)

Therefore, option D is the correct


answer.
148. Ans. C.
1) Z lives on the fifth floor.
2) The one, who works in ONGC, lives on
the topmost floor.
3) The one, who works in Tata motors,
lives just above X.

50
11) Z and X don’t work in SBI. 7) The one, who works in HPCL, lives
(Here, case 2 will be eliminated) above T.
8) There is only one floor between U and
the one, who works in RIL.
9) There are three floors between T and
the one, who works in IOC.
10) Y works in BPCL.

Therefore, option C is the correct


answer.
149. Ans. D.
1) Z lives on the fifth floor.
2) The one, who works in ONGC, lives on
the topmost floor.
3) The one, who works in Tata motors,
lives just above X.
4) V lives on an even numbered floor.
5) There are two floors between the
floors on which X and V lives.

11) Z and X don’t work in SBI.


(Here, case 2 will be eliminated)

Therefore, option D is the correct


answer.
150. Ans. D.
1) Z lives on the fifth floor.
6) Neither X nor Z works in RIL. 2) The one, who works in ONGC, lives on
the topmost floor.

51
3) The one, who works in Tata motors,
lives just above X.
4) V lives on an even numbered floor.
5) There are two floors between the
floors on which X and V lives.

11) Z and X don’t work in SBI.


(Here, case 2 will be eliminated)

6) Neither X nor Z works in RIL.


7) The one, who works in HPCL, lives
above T.
8) There is only one floor between U and
the one, who works in RIL.
9) There are three floors between T and
the one, who works in IOC.
10) Y works in BPCL. Therefore, option D is the correct
answer.

52

You might also like